You are on page 1of 59

Dosier módulo VI

Módulo VI
1. Unidad I: Principios de Conteo 6
1.1. ¿QUÉ ES COMBINATORIA Y QUE ESTUDIA?
1.2. PRINCIPIO DE LA SUMA
1.3. PRINCIPIO DE LA MULTIPLICACIÓN
1.4. PRINCIPIO DE CORRESPONDENCIA
1.5. PRICINPIO DE INCLUSIÓN-EXCLUSIÓN
1.6. RECURRENCIA
1.7. PRINCIPIO DE LAS CASILLAS DE DIRICHLET
1.8. PROBLEMAS

2. Unidad II: Combinaciones y el número combinatorio 20


2.1. MODELO DE CONJUNTOS
2.2. MODELO DE CAMINOS
2.3. CADENAS DE CEROS Y UNOS
2.4. TRIÁNGULO DE PASCAL
2.5. BINOMIO DE NEWTON
2.6. PROBLEMAS

3. Unidad III: Permutaciones y Arreglos 36


3.1. PERMUTACIONES
3.2. PERMUTACIONES CIRCULARES
3.3. PERMUTACIONES CON REPETICIÓN
3.4. ARREGLOS
3.5. PROBLEMAS

4. Unidad IV: Extensiones del número combinatorio 43


4.1. SEPARADORES
4.2. MULTICOMBINATORIO
4.3. PROBLEMAS
4.4. PRINCIPIO DE
- INCLUSIÓN EXCLUSIÓN
4.5. DESÓRDENES
4.6. RECURRENCIA
4.7. PRINCIPIO DE CASILLAS
4.8. PROBLEMAS
.

Una manera muy sencilla de definir a la Combinatoria es entenderla como aquella área de la matemá ca
que trata el problema de contar Esta ac vidad tan natural y a la que nos aproximamos desde edades
muy tempranas, ene dos caracterís cas centrales:

1. Para contar, hay que considerar todas las posibilidades.

2. Y además, hay que asegurarse que cada objeto de conteo se cuenta exactamente una vez, es decir,
hay que evitar contar dos o más veces a un mismo objeto.

Es decir, cada objeto se cuenta al menos una vez y a lo sumo una vez. Sí, esto es evidente, pero pronto
podrá darse cuenta que esto puede resultar muy complicado de percibir.

Para contar hay muchas formas, y la más elemental es hacer un conteo exhaus vo, es decir, uno por
uno elaborando un listado completo de los objetos, o lo que es lo mismo, un censo. Ahora bien, si lo
que nos proponemos contar es la can dad de números de un millón de cifras que comienzan con el
dígito 4, obviamente, no haremos un censo, nunca terminaríamos; allí entra en juego la combinatoria, la
combinatoria proporciona métodos y técnicas para resolver este y otros problemas en el que el conteo
exhaus vo no funciona. Por tal mo vo, algunos consideran a la combinatoria como el arte de ``contar
sin contar''.
Principios de conteo
En este apartado desarrollaremos algunos principios básicos de conteo y enumeración. Cuando nos
interesamos en determinar el número de elementos en un conjunto dado, estamos en un caso de conteo;
mientras que cuando nos interesa listar los elementos estamos en el caso de enumeración. Ambos
problemas son importantes; hay situaciones en las que nos interesa no sólo saber cuántos elementos
hay en un conjunto dado, si no además saber cuáles son tales elementos, de aquí que con frecuencia los
métodos de conteo y enumeración son inseparables. El propósito de este apartado es el de desarrollar
algunas técnicas fundamentales de conteo en los que la enumeración no aparece de manera explícita;
se basan fundamentalmente en algunos principios cuya simplicidad con frecuencia impide valorar su
potencia; desarrollar la habilidad de aplicar correctamente tales principios requiere alguna prác ca por
lo que se proponen ejercicios diversos para la aplicación de los mismos en diferentes contextos.

1.1. ¿QUÉ ES COMBINATORIA Y QUE ESTUDIA?


La Combinatoria es la parte de la Matemá ca que estudia las diversas formas de realizar agrupaciones
con los elementos de un conjunto, formándolas y calculando su número. Existen dis ntas formas de
realizar estas agrupaciones, según se repitan los elementos o no, según se puedan tomar todos los
elementos de que disponemos o no y si influye o no el orden de colocación de los elementos. El desarrollo
de la combinatoria está fuertemente ligado con su aplicación en la teoría de la probabilidad, pero también
es importante en otras ciencias como la informá ca, por ejemplo en la teoría de la codificación y en el
análisis de algoritmos.

El objeto del Análisis Combinatorio o Combinatoria es el estudio de las dis ntas ordenaciones que pueden
formularse con los elementos de un conjunto, de los dis ntos grupos que pueden formarse con aquellos
elementos y de las relaciones entre unos y otros grupos

1.2. PRINCIPIO DE LA SUMA


Un concepto fundamental cuando se trata de contar, es el de cardinal de un conjunto. En el caso de los
conjuntos finitos, el cardinal de un conjunto A es simplemente el número de elementos que posee y se
acostumbra denotarlo por |A| o bien por Car d(A).

La técnica más elemental para contar los elementos de un conjunto es la de separar sus elementos
en clases disjuntas, de forma tal que su reunión incluya todos los elementos del conjunto. En otras
palabras se requiere que cada elemento del conjunto debe pertenecer a una sola de las clases y que
todo elemento del conjunto pertenece a una de las clases en las que se separa el conjunto.

Definición 1.1. Un conjunto finito A ha sido separado en n clases disjuntas A1 , A2 , A3 , . . . , An , si se


sa sface simultáneamente:

. Ai ∩ Aj = ∅ para todo i ̸= j.

. ni=1 Ai = A.
7
También decimos que el conjunto A ha sido par cionado y las clases disjuntas (o exlcuyentes) se denominan
los elementos de la par ción.

Teorema 1.1. Principio de la Suma: Si el conjunto A es posible separarlo en clases A1 , A2 , . . . , An , el total


de elementos de A, es igual a la suma de los cardinales de cada una de las clases. Es decir

|A1 | + |A2 | + · · · + |An | = |A|

El principio de la suma suele ser enunciado también de la siguiente forma: si un suceso A puede ocurrir de
n maneras, un suceso B puede ocurrir de m maneras y ambos sucesos no pueden ocurrir simultáneamente,
entonces el suceso A o B puede ocurrir de (n+m) formas. Por supuesto ene su versión cuando hay más
de dos sucesos, con la condición obvia que dos cualesquiera de ellos no pueden ocurrir simultáneamente.

1.3. PRINCIPIO DE LA MULTIPLICACIÓN


Cuando se trata de contar parejas (x, y ) conociendo el número de opciones de cada una de las componentes
del par, se u liza el conocido como principio de la mul plicación que afirma que el total de posibles
pares que se pueden formar es el producto del número de alterna vas que se dispone para la primera
componente por el número de alterna vas para la segunda componente.

Por ejemplo si se lanza al aire un dado dos veces y anotamos los posibles resultados, estos los podemos
registrar mediante un par (x, y ) registrando en la primera componente el resultado de la primera rada y
en la segunda componente el resultado de la segunda rada. Siendo que en cada rada hay seis posibles
resultados: 1, 2, 3, 4, 5, 6, el total de posibles pares es 36. En efecto para cada uno de los posibles
resultados para la primera rada tenemos 6 posibles resultados para la segunda rada y siendo que
hay seis posibles resultados en la primera rada, el total de resultados será de 36.

De manera completamente análoga suponga que deseamos determinar el total de secuencias de tres
letras, es decir ternas (x, y , z) que se pueden formar con las letras a, b, c, d, e, f , de forma tal que no se
permite la repe ción de letras. Habiendo 6 opciones de letras por colocar en la primera posición, para la
segunda posición sólo tendremos 5 opciones puesto que no se acepta la repe ción de letras en la terna;
restricción que nos deja en la tercera posición sólo con 4 letras como posibles opciones. Así el total de
ternas con la restricción planteada será de 6 · 5 · 4 = 120.

Teorema 1.2. Principio del Producto: Si A1 , A2 , A3 , . . . , Ak es una sucesión de conjuntos con cardinales
n1 , n2 , n3 , . . . , nk respec vamente, entonces el conjunto que se ob ene haciendo el producto cartesiano
de tales conjuntos ene por cardinal el producto de los cardinales de los conjuntos dados. Es decir

|A1 × A2 × · · · × Ak | = n1 n2 · · · nk

En par cular, cuando se trata del mismo conjunto A de cardinalidad n en cada uno de los factores, se
ob ene
k v eces
z }| {
| A × A × · · · × A | = nk
8
En otra versión del principio de la mul plicación, éste se presenta en la forma siguiente: si un conjunto
puede ser par cionado en k clases y cada una de las clases puede ser separada en t pos de elementos,
el total de pos de elementos es kt.

Otra versión muy frecuente del principio de la mul plicación se presenta de la forma siguiente: si hay
n1 alterna vas de seleccionar un primer objeto y para seleccionar un segundo objeto se dispone de n2
formas, la selección del par ordenado de objetos puede ser realizada en n1 n2 formas.

Analice el siguiente problema: Si desde Santa Ana a San Salvador se puede viajar por 3 rutas dis ntas de
autobus, y de San Salvador a San Miguel se puede viajar por 5 rutas dis ntas de autobus, ¿de cuántas
formas dis ntas se puede viajar en autobus desde Santa Ana hasta San Miguel pasando por San Salvador?
¡Inténtelo!

1.4. PRINCIPIO DE CORRESPONDENCIA


Consideremos el problema siguiente: En un campeonato de fútbol se enfrentan n equipos. En cada ronda
los equipos perdedores son eliminados. Si en una ronda el número de equipos aún par cipando es impar,
uno de los equipos, elegido mediante sorteo, descansa y pasa a la ronda siguiente. ¿Cuántos juegos se
realizan durante el campeonato?

Este problema ene por supuesto muchas maneras de ser abordado, pero en este caso vamos a destacar
la forma que se apoya en el denominado principio de correspondencia.
Definición 1.2. Dos conjuntos A y B son tales que a cada elemento de A se le puede asociar uno y sólo
un elemento de B y viceversa (a cada elemento de B se le puede asociar uno y sólo uno de los elementos
de A), entonces decimos que A y B pueden ponerse en correspondencia biyec va. En este caso se dice
también que los conjuntos A y B son coordinables y escribimos en forma simbólica A ∼= B.3
Teorema 1.3. Principio de Correspondencia: Si A y B son coordinables entonces enen el mismo número
de elementos o cardinal.
Veamos cómo se aplica el principio anterior al problema propuesto: Como en cada par do hay un perdedor
y sólo uno y para cada perdedor hay uno y un único par do, para contar el número de par dos nos basta
contar el número de perdedores y siendo que al final del campeonato sólo queda uno de los equipos,
el número de perdedores es n − 1, que debe por supuesto ser el número de par dos jugados en el
campeonato. Así, este número debe ser n − 1.

El principio anterior es importante cuando la tarea de contar los elementos de un conjunto A resulta
más di cil que la de contar en un conjunto B que puede ponerse en correspondencia biyec va con A.
Se cuenta entonces en el conjunto B y automá camente se ha contado en el conjunto A.

He aquí otro ejemplo en donde se aplica el principio de correspondencia biyec va: Queremos determinar
el total de formas que tenemos de seleccionar 9 elementos en un conjunto que posee 10 elementos.
3
Un conjunto A se dice que es finito si es vacío o si es coordinable con un conjunto de la forma {1, 2, 3, . . . , n} y en este
úl mo caso se dice que el cardinal de A es n, o bien que el número de elementos que posee es n. Cuando A es vacío decimos
que el conjunto ene cero elementos.
9
Siendo que hay una correspondencia biyec va entre los conjuntos de 9 elementos con los conjuntos de
1 elemento, es suficiente para resolver el problema contar el número de selecciones de 1 elemento, que
son por supuesto 10 formas.

⋆⋆⋆
Analicemos ahora algunas ejemplos un poco más complicados que u lizan estos tres principios fundamentales;
con frecuencia, en un mismo problema se u liza más de uno.

EJEMPLO 1.1
Un profesor ene 35 estudiantes en el curso de Álgebra y 38 estudiantes en el curso de Geometría.
¿Cuántos estudiantes ene en total?

La respuesta 73 estudiantes sólo sería válida en el caso de que ningún estudiante reciba los dos cursos
impar dos por el profesor. Si hay estudiantes que reciben ambos cursos, el conteo exigiría que se separe
el total de estudiantes en las tres clases disjuntas: los que reciben sólo el curso de Álgebra, los que reciben
sólo el curso de Geometría y los que reciben ambos cursos. Suponga por ejemplo que hay 10 estudiantes
que reciben ambos cursos, entonces los conjuntos no serían disjuntos; si descontamos los estudiantes
comunes del curso de Álgebra y Geometría, tendríamos 25 que reciben sólo el curso de Álgebra y 28
estudiantes que reciben sólo el curso de Geometría. Así el total es la suma del número de elementos
que poseen los tres conjuntos disjuntos, es decir: 25 + 28 + 10 = 63.

EJEMPLO 1.2
Se enen 6 libros dis ntos de Álgebra, 5 libros dis ntos de Geometría, y 4 libros dis ntos de Trigonometría.
¿De cuántas formas es posible seleccionar un par no ordenado de libros que no sean de la misma
asignatura?

Siendo que los libros seleccionados deben ser de asignaturas diferentes, las posibilidades de combinar
son: Un libro de Álgebra y uno de Geometría; uno de Álgebra y uno de Trigonometría y por úl mo uno
de Geometría y uno de Trigonometría; con ello hemos separado los pares posibles en casos disjuntos.
Aplicando en cada uno de los casos el principio de la mul plicación tenemos que las alterna vas para
cada caso son: 6 · 5, 6 · 4 y 5 · 4, respec vamente. Así, por el principio de la suma, el total de alterna vas
para hacer la selección del par de libros es: 30 + 24 + 20 = 74.

EJEMPLO 1.3
Se consideran las cadenas de ceros y unos de longitud 5, es decir secuencias con cinco caracteres entre
los dígitos cero y uno. Por ejemplo la cadena 00010 es de longitud cinco. ¿Cuál es el total de cadenas de
ceros y unos de longitud 5?

Observe que si P = {0, 1} , cada una de las cadenas puede ser iden ficada mediante el quinteto
(x1 , x2 , x3 , x4 , x5 ) del producto cartesiano P × P × P × P × P = P 5 , recíprocamente, todo elemento del
producto cartesiano P 5 está asociado a una cadena de ceros y unos de longitud cinco. En consecuencia,
dada esta correspondecia biyec va, el total de cadenas será |P 5 | = 25 = 32.

10
EJEMPLO 1.4
¿Cuál es el número de subconjuntos que posee un conjunto A de cardinal n?

Como en el ejemplo anterior, nos apoyaremos en la correspondencia biyec va entre los subconjuntos
de A con las cadenas de longitud n. Supongamos ordenados los elementos de A del primero al n-ésimo
elemento y adoptemos la convención siguiente: a cada subconjunto de A asociamos la cadena de longitud
n, cuya componente en la posición k-ésima es 0 si el elemento k-ésimo de A no pertenece al subconjunto,
o bien 1 si el elemento pertenece al subconjunto. Por ejemplo la cadena (0, 0, 0, 0, . . . , 0) está asociada
al subconjunto vacío; el subconjunto con sólo el primer elemento de A se asocia con la cadena (1, 0, 0, 0, . . . , 0)
que posee sólo ceros salvo en la primera componente. Con esta convención se logra en efecto la correspondencia
biyec va y podemos, en vez de contar subconjuntos de A, contar las cadenas de ceros y unos de longitud
n, que como extensión del ejemplo anterior será |P n | = 2n (u lizamos el mismo conjunto P del ejemplo
anterior).

EJEMPLO 1.5
Usando las letras a, b, c, d, e, ¿de cuántas formas es posible formar una secuencia ordenada de tres
letras en cada uno de los siguientes casos?

a) Si la repe ción de letras está permi da.

b) Si no se permite la repe ción de letras.

c) Sin repe ción de letras y que la secuencia tenga a la letra e.

d) Con repe ción de letras conteniendo e.

a) En el primero de los casos como la repe ción está permi da, el problema es equivalente a determinar
el número de ternas del conjunto A3 , en donde A = {a, b, c, d, e}, que es 53 = 125.

b) En el segundo caso no se permite la repe ción, se debe tomar en cuenta que para formar las ternas
solicitadas, para la primera componente de la terna disponemos de todos los elementos del conjunto
A; sin embargo para la segunda componente, disponemos de un conjunto con cuatro elementos
puesto que debemos excluir el elemento colocado en la primera componente; de manera similar para
la tercera componente sólo dispondremos de un conjunto con tres elementos habiendo excluido los
pertenecientes a la primera y segunda componente de la terna. Así, el total de ternas con la condición
solicitada es: 5 · 4 · 3 = 60.

c) Para el tercer caso, en el que la letra e está obligada a pertenecer a la secuencia, podemos dividir en
casos de acuerdo a la posición que ocupe la letra e en la terna, que son obviamente casos disjuntos.
Así, si ocupa la primera posición las ternas son de la forma (e, x2 , x3 ) y para la segunda componente
se dispone de cuatro elementos en tanto que para la tercera componente se dispone nada más que
de tres elementos, por lo que en este caso tendremos 4 · 3 = 12 posibilidades. Los otros casos a
considerar son las ternas de la forma (x1 , e, x3 ) y las ternas de la forma (x1 , x2 , e) que son igualmente
12 posibilidades en cada uno, por lo que en total tendremos 36 secuencias con la condición dada.

d) Para el úl mo caso en el cual se permite repe ción pero la secuencia con ene obligatoriamente a la e,
haremos la separación de ternas en las clases siguientes: (e, x2 , x3 ), (x1 , e, x3 ) y (x1 , x2 , e), tomando
11
en cuenta que en la primera clase tendremos para x2 y x3 cinco posibilidades, es decir tendremos 25
ternas en esta primera clase; en la segunda clase habrá que excluir x1 = e, puesto que las ternas
con e en la primera componente ya fueron contadas en las ternas de la primera clase, así, en esta
segunda clase de ternas tendremos 4 posibilidades para la primera componente y 5 para la tercera
componente, es decir tendremos 20 ternas en esta clase; en la tercera clase debemos excluir los casos
x1 = e y x2 = e puesto que las ternas de este po ya fueron consideradas en las dos clases anteriores
y en consecuencia tendremos 4 posibilidades para x1 y 4 posibilidades para x2 , resultando 16 ternas
de esta clase. En total tendremos 25 + 20 + 16 = 61 ternas. ¡Recuerde que contar significa tomar
en cuenta todos los elementos del conjunto pero cada elemento tomado en cuenta sólo una vez!

EJEMPLO 1.6
Considere una cuadrícula 8×8. Determine el número de cuadrados formados por vér ces de la cuadrícula
cuyos lados son paralelos a los segmentos de la cuadrícula.

Este es un pico problema en el que se evidencia la importancia de contar con método, de lo contrario
la tarea se vuelve complicada. Tal como lo demanda el principio de la suma, lo mejor es separlos en
clases disjuntas. Así, contaremos por separado los cuadrados de lado 1, los cuadrados de lado 2 y así
sucesivamente hasta los cuadrados de lado 8.
Los cuadrados de lado 1 son 8 por cada fila y siendo que tenemos 8 filas, el total de cuadrados de lado
1 es 8 · 8 = 64. Para los cuadrados de lado 2 observe que para formarlos se requiere un par de filas
consecu vas y en ellas los cuadrados de lado 2 que pueden formarse son 7; si se toma en cuenta que
se pueden formar siete pares de filas consecu vas se ob ene que el total de cuadrados de lado 2 es
7 · 7 = 49. De forma completamente análoga para formar cuadrados de lado 3 requerimos disponer de
tres filas consecu vas y en ellas es posible construir 6 cuadrados de lado 3; como se pueden formar seis
tríos de filas consecu vas, el número de cuadrados de lado 3 será de 6 · 6 = 36 cuadrados. Después de
considerar los casos anteriores los siguientes casos son fáciles de deducir: el total de cuadrados de lado
4 será 5 · 5, el de cuadrados de lado 5, será 4 · 4, los de lado 6, serán 3 · 3, los de lado 7 serán 2 · 2 y los de
lado 8, 1·1. Así el total de cuadrados buscado será 1·1+2·2+3·3+4·4+5·5+6·6+7·7+8·8 = 204
cuadrados.

EJEMPLO 1.7
Determine el número de diagonales que posee un decágono.

Por supuesto que dibujar las diagonales y después contarlas es una de las opciones para resolver el
ejercicio, pero obviamente es una solución poco prác ca y di cil de generalizar (y también di cil porque
el dibujo puede complicarse demasiado). Vamos a u lizar la separación de las diagonales en casos disjuntos
para en seguida aplicar el principio de la suma. Recuerde que en un polígono se denomina diagonal al
segmento que une dos vér ces no consecu vos del polígono. Para separar en clases disjuntas vamos a
considerar las diagonales que “salen” de cada uno de los vér ces. Dado que en un decágono cada vér ce
ene 7 vér ces no consecu vos son siete diagonales que salen de cada vér ce; como en un decágono
tenemos 10 vér ces, el total de diagonales será de 35 tomando en cuenta que cada diagonal ha sido
contada dos veces, una vez por cada extremo de la diagonal.
12
1.5. PRICINPIO DE INCLUSIÓN-EXCLUSIÓN
Clasificar, como ya se habrá observado, es un método importante cuando se trata de contar objetos.
Supone sin embargo que somos capaces de par cionar el conjunto dado en clases disjuntas y esto no
siempre es sencillo. Es necesario desarrollar otros métodos.

Una estrategia que en ciertos casos ofrece alguna ventaja, es la de usar el criterio de contar en el
complemento, en vez de contar directamente en el conjunto dado. Los dos ejemplos siguientes ilustran
este caso.

EJEMPLO 1.8
De las cadenas binarias de ceros y unos, de longitud 10, ¿cuántas enen la propiedad de poseer por lo
menos un 0?

EJEMPLO 1.9
Cinco niños solicitan, en una sorbetería que dispone de sorbetes de 8 diferentes sabores, un sorbete
cada uno. De todas las posibles solicitudes que pueden hacer, ¿en cuántas de ellas los sabores elegidos
por los niños, coinciden en cuando menos dos de ellos?
En la solución de los dos problemas anteriores aún cuando es posible clasificar en conjuntos disjuntos
de manera directa, el número de clases que resulta es demasiado alto y es preferible contar por su
complemento.

Hay ocasiones sin embargo en las que la clasificación en el conjunto o en su complemento ofrece el
mismo nivel de dificultad y en este caso el principio de inclusión-exclusión resulta ser un valioso recurso
de conteo.

En su versión más elemental, el principio de inclusión- exclusión nos dice que el número de elementos
en la reunión de dos conjuntos A y B, se calcula haciendo la suma de los números de elementos A mas
los de B y se resta el número de elementos comunes a ambos conjuntos, es decir los elementos de la
intersección de ambos conjuntos. La interpretación es obvia: la resta del número de elementos de la
intersección de los conjuntos evita que éstos se cuenten dos veces por cuanto están incorporados como
elementos del conjunto A y como elementos del conjunto B. En expresión conjun sta, tenemos:

|A ∪ B| = |A| + |B| − |A ∩ B|

En el caso de la reunión de tres conjuntos A, B, C, los ajustes que debemos realizar para calcular el
número de elementos de la reunión de los tres conjuntos son menos obvios. En este caso, si sumamos el
número de elementos de A, B y C, observamos que los elementos comunes a dos de tales conjuntos han
sido contados dos veces, mientras que los elementos comunes a los tres conjuntos han sido contados
tres veces. Procedemos a corregir, restando los elementos comunes a dos de los conjuntos dados; en
este caso tenemos tres posibilidades A con B; A con C; B con C. Con esta corrección los elementos
estrictamente pertenecientes a dos de los conjuntos dados quedan contados sólo una vez; sin embargo,
los elementos comunes a los tres conjuntos, que anteriormente habían sido contados tres veces, ahora
han sido descontados tres veces, lo que hace que en suma no hayan sido contados. Hacemos ahora
la corrección final sumando el número de elementos que son comunes a los tres conjuntos. Hemos
13
obtenido en consecuencia el resultado siguiente:

|A ∪ B ∪ C| = |A| + |B| + |C| − (|A ∩ B| + |B ∩ C| + |C ∩ A|) + |A ∩ B ∩ C|

1.6. RECURRENCIA.
La recursión es un poderoso método de análisis que consiste en suponer que se conocen los valores
de una función de conteo para todos los valores previos a n, e intentar determinar, a par r de tal
información, el valor de tal función para el valor de n.

Cuando se trata de contar objetos en función de una o más variables, la condición ideal es la de lograr una
fórmula explícita que relacione tales variables. En ciertas ocasiones, sin embargo, tal condición resulta
di cil de alcanzar y con frecuencia es suficiente obtener las suficientes relaciones que nos permitan
generar un algoritmo de cálculo para la resolución de nuestro problema en el caso n-ésimo, a par r de
la información acerca de los casos previos, es decir buscar una recurrencia; en otras ocasiones incluso
es preferible y más sencillo buscar tales relaciones. He aquí algunos de ejemplos de cómo esta técnica
permite resolver algunos problemas:

EJEMPLO 1.10
Determinar el número de subconjuntos de un conjunto finito dado.

Podemos determinar el número de subconjuntos de un conjunto finito dado a par r de una recurrencia.
En efecto, el número de subconjuntos de un conjunto unitario es 2, y si denotamos el número de subconjuntos
de un conjunto con n elementos como Sn , el número de subconjuntos en el caso de un conjunto con (n+
1) elementos puede ser calculado de la siguiente manera: consideremos el úl mo elemento incorporado,
entonces sólo hay dos posibilidades de subconjuntos, los que no incluyen el elemento úl mo que son
en total Sn , y los que sí lo incluyen, que se ob enen introduciendo éste en cada uno de los subconjuntos
de los n primeros elementos, que de nuevo son Sn . Hemos demostrado así que:

Sn+1 = 2Sn
Esta relación junto con la condición inicial S1 = 2, permite calcular todos los valores de Sn , de manera
recurrente, para cualquier número natural n

Sn = 2Sn−1
= 2 (2Sn−2 ) = 22 Sn−2
= 22 (2Sn−3 ) = 23 Sn−3
..
.
= 2n−1 S1 = 2n

EJEMPLO 1.11
Determinar el número de diagonales que posee un polígono convexo de n lados.

14
Aún cuando no podamos determinar tal número, podemos intentar establecer una relación entre el
número de diagonales de un polígono de n lados con las de un polígono de (n + 1) lados. Sea Dn
el número de diagonales de un polígono de n lados. De lo que se trata es de establecer una relación
entre Dn y Dn+1 . Si agregamos un vér ce a un polígono de n lados, éste lo podemos imaginar ubicado
entre dos de los n vér ces an guos. Este vér ce puede ser conectado con n − 2 vér ces an guos para
formar diagonales; así se formarán n − 2 nuevas diagonales; sin embargo, un lado en el an guo polígono
(justamente el lado que unía los dos vér ces vecinos al nuevo vér ce) se ha transformado en diagonal;
en otras palabras el nuevo vér ce ha generado (n − 1) nuevas diagonales. Concluimos entonces que
debe cumplirse la relación de recurrencia:

Dn+1 = Dn + (n − 1)
Por otra parte, observe que D3 = 0. La información anterior es suficiente para calcular el número
de diagonales de cualquier polígono, simplemente aplicando de manera itera va la relación anterior.
¡Inténtelo!

1.7. PRINCIPIO DE LAS CASILLAS DE DIRICHLET .


El principio de las casillas de Dirichlet, también conocido como el principio del palomar o principio de los
cajones, es un principio fácil de aceptar en su versión más elemental. Su aplicación invade ámbitos muy
diversos y con frecuencia su u lización permite resolver problemas que distan mucho de ser triviales.

La idea central del principio de casillas es la siguiente: si se enen n + 1 bolas que serán colocadas en
n contenedores (las casillas), entonces forzosamente hay un contenedor que tendrá dos o más bolas,
¡lógico! ¿cierto? A con nuación se muestran otras versiones, pero que en escencia son lo mismo

Teorema 1.4. Principio de Casillas:


Versión 1: Si más de n bolitas se encuentran distribuidas en n cajas entonces hay una caja que con ene
al menos 2 bolitas.
Versión 2: Si más de mn bolitas se encuentran distribuidas en n cajas entonces hay una caja que con ene
al menos m + 1 bolitas.
S
Versión 3: Si la suma de n can dades es mayor que S entones una de las can dades es mayor que .
n
Versión 4: Un segmento I de longitud t con ene varios segmentos cuyas longitudes suman más que t,
entonces al menos dos de los segmentos contenidos en I se solapan.
Versión 5: Un segmento I de longitud t con ene varios segmentos cuyas longitudes suman más que kt,
donde k es un entero posi vo, entonces hay un punto del segmento I que está contenido en k + 1 de los
segmentos contenidos en I.
También hay versiones análogas a las úl mas dos para figuras planas y áreas. Versión 6: Sean q1 , q2 , . . . , qn
enteros posi vos. Si q1 + q2 + · · · + qn − n + 1 objetos son colocados en n cajas, llamadas C1 , C2 , . . . , Cn ,
entonces al menos una caja Ci ene qi objetos o más.

EJEMPLO 1.12
De un conjunto de 12 números enteros de dos dígitos, siempre podemos seleccionar dos de ellos cuya
15
diferencia ene la forma aa.4

Obsérvese primero que los números de la forma aa son todos los múl plos de 11 de dos cifras. Ahora
bien, si hacemos la división de los números dados por 11, los posibles residuos son 0, 1, 2, . . . , 10, y
como se escogieron 12 números, hay por lo menos una pareja de números que enen el mismo residuo,
entonces su diferencia será divisible por 11 y deberá ser de la forma aa.
Lectura Complementaria: Eder Alexander Jacobo. Material de Autoformación Docente para Matemá ca
9° grado. Lección 5. Principios Básicos de Conteo. MINED

1.8. PROBLEMAS
1. Cinco jueces de un deporte determinado disponen de una cartulina en la que por un lado hay un
1 y por el otro un 0. ¿Cuántas configuraciones dis ntas pueden generar?

2. Un marino ene 4 banderas dis ntas para hacer señales. ¿Cuántas señales diferentes puede hacer
si coloca 3 banderas en un más l una sobre otra?

3. El ascensor de un edificio de 5 plantas se pone en marcha con 3 pasajeros. ¿De cuantos modos
dis ntos se pueden distribuir los pasajeros entre los pisos?

4. Se lanzan dos dados y se observa la suma de los puntos obtenidos. ¿En cuántos casos el resultado
es divisible por tres?

5. En cierto lenguaje de programación un nombre de una variable puede formarse con una letra o
bien una letra seguida de un un dígito. ¿Cuántas variables diferentes pueden formarse? Asuma
para este ejercicio 27 letras.

6. Se ene una ra o banda formada por n rectángulos iguales formando una fila. Cada rectángulo
puede colorearse de blanco o negro.

a) ¿Cuántas configuraciones diferentes se pueden hacer?


b) ¿Cuántas formas dis ntas tenemos de colorear la ra, de modo que se obtenga un patrón
simétrico?
c) ¿Y si se desea con colores alternados?

7. ¿De cuantas maneras pueden colocarse un alfil blanco y uno negro en un tablero de ajedrez de
modo que se ataquen mutuamente?

8. ¿De cuántas formas es posible seleccionar dos cartas diferentes de una baraja de 52 cartas de
forma tal que la primera carta sea un as y la segunda no sea una reina? ¿y si se demanda es que la
primera carta sea de espadas y la segunda no sea reina?
4
La notación ab hace referencia a un número cuya cifra de las unidades es a y la cifra de las decenas es b, y la se coloca la
raya encima para evitar confundir este número con ab, que representa el producto de a por b. Esta claro que esta notación
puede u lizarse para representar números de tres, cuatro o más cifras.

16
9. ¿Cuántos números enteros hay entre 1000 y 10000 con la condición de que sus dígitos sean
diferentes? ¿y si se permite la repe ción de dígitos, pero no se permiten los dígitos 2 ó 4? ¿y
cuando los dígitos deben ser dis ntos y al menos uno de los dígitos 2 y 4 debe aparecer?

10. Con los dígitos del 0 al 9, ¿cuántas secuencias de longitud 5 pueden ser formadas de forma tal que
aparezcan exactamente dos de los diez dígitos?

11. De cuántas formas pueden ser colocadas en un tablero de ajedrez dos torres idén cas en una fila
común o en una columna común? ¿y si las torres no son idén cas?5

12. ¿Cuántos números de cuatro dígitos se pueden formar con los dígitos 1, 2, 3, 4, 5 (con posible
repe ción) tales que sean divisibles por 4?

13. ¿Cuántos cuadrados de longitud entera pueden formarse en una cuadrícula de n × n (con los lados
formados por segmentos de la cuadrícula)?

14. ¿Cuántas diagonales se pueden trazar en un polígono convexo de n lados?

15. Se dice que en un polígono se ha realizado una triangulación, cuando el interior del polígono se
cubre con triángulos formados con vér ces del polígono de forma tal que los triángulos no tengan
en común más que conjuntos de puntos de área cero. ¿Cuántos triángulos se requieren en una
triangulación en un polígono de n lados?

16. ¿Cuántas secuencias de tres letras diferentes pueden ser formadas haciendo uso de las letras
a, b, c, d, e, f en las cuales aparecen la letra e, o la letra f , o ambas e y f ?

17. Cuando se listan los números del 1 al 10000, ¿cuántas veces se hace uso del dígito 5? ¿y cuántas
veces aparece el ``25''?

18. En un tablero de ajedrez, de cuántas formas se pueden colocar dos reinas de forma tal que no
estén ni en la misma fila, ni en la misma columna ni en la misma diagonal?6

19. ¿Cuántos números posi vos pueden formarse como suma de los números 1, 3, 5, 10, 20 y 50?

20. Considere una cuadrícula de 5 × 5 ¿cuántos rectángulos podemos formar con los ver ces de dicha
cuadrícula cuyos lados sean paralelos a los segmentos de la cuadrícula?

21. Una persona desea invitar a uno o más de sus 10 amigos a su fiesta de cumpleaños. ¿de cuántas
formas podría hacer tal invitación?

22. ¿Cuántos números de tres cifras dis ntas pueden formarse con los dígitos impares?

23. Un turista debe trasladarse de una ciudad a otra. Para hacerlo pude optar viajar por avión, autobus
o tren, y en cada uno de estos medios puede elegir viajar en 1a clase o en clase turís ca. ¿De
cuántas maneras dis ntas puede realizar el viaje?

24. En una fiesta se encuentran 10 hombres y 8 mujeres. ¿De cuántas maneras pueden integrarse en
parejas para bailar una determinada pieza?
5
Cuando un par de torres de ajedrez comparten fila o columna, decimos que las torres se están ``atacando''.
6
Las reinas no se están ``atacando''.
17
25. Un menú turís co permite seleccionar una entrada de entre cuatro posibles, una comida caliente
de entre tres, y un postre de entre cinco. ¿De cuántas formas puede elegir su menú un turista si
desea que el pollo guisado y el flan no aparezcan en el mismo menú?

26. ¿Cuántos boletos capicuas de 5 cifras hay?

27. Seis matrimonios posan en fila para una fotogra a. ¿De cuántas maneras pueden ubicarse si los
miembros de cada pareja deben aparecer juntos?

28. Se desea colocar 9 libros dis ntos en tres estantes en una biblioteca, 3 en cada estante. ¿De
cuántas pueden ubicarse?

29. Al ingresar a un colegio, 6 chicas debe optar entre asis r a las clases de inglés o a las de francés,
¿de cuántas maneras pueden repar rse si Paula y Daniela deciden elegir la misma materia y Cecilia
no quiere compar r la elección de ellas?

30. Un sábado, cuando iban de compras, Juana y Teresa vieron a dos hombres alejarse en automóvil de
la fachada de una joyería, justo antes de que sonará una alarma contra robos. Aunque todo ocurrió
muy rápido, cuando fueron interrogadas las dos jóvenes, pudieron dar a la polica la siguiente
información acerca de la placa (que consta de 2 letras seguidas de 4 dígitos) del automóvil que
huyó: Teresa estaba segura de que la segunda letra de la placa era una O o una Q, y el el úl mo
dígito era un 3 ó un 8. Juana dijo que la primera letra de la placa era una C o una G y que el primer
dígito era defini vamente un 7. ¿Cuántas placas diferentes tendrá que verificar la policía?

31. ¿Cuántos números naturales menores o iguales a un millón no enen dos cifras consecu vas
iguales?

32. ¿De cuántas maneras se pueden colocar 8 torres en un tablero de ajedrez tal que no se ataquen?

33. ¿De cuántas maneras se puede escoger un cuadrado negro y uno blanco de un tablero de ajedrez
tal que no pertenezcan a la misma fila ni a la misma columna?

34. Pruebe que el número máximo de alfiles (todos iguales) que se pueden colocar en un tablero
cuadrado de n × n casillas sin que haya dos que se ataquen es 2n − 2, y que el número de estas
configuraciones es 2n .

35. Entre 20000 y 70000, encuentre el número de enteros pares en los cuales no se repita ningún
dígito.

36. Una emisora de radio dispone de seis programas para cubrir las dos horas de mayor audiencia.
Dos de estos programas enen una duración de una hora y los cuatro restantes de 30 minutos,
siendo todos los programas dis ntos. ¿De cuántas maneras puede la emisora cubrir las dos horas
de programación?

37. ¿Cuántos números del 1 al 100000 no son divisibles ni por cinco ni por siete?

38. Tenemos los números del 100000 al 999999. ¿Cuántos de ellos cumplen que enen al menos un
dígito cero o al menos un dígito uno?

18
39. Dado el conjunto de dígitos {1, 3, 6, 7, 9}, determine el número de formas de formar números de
4 dígitos tales que sean múl plos de 3.

40. ¿Cuántos divisores posi vos ene el número 121 0?

41. María y Laura idean el siguiente juego: cada una lanza un dado, si en los dados sale el mismo
número, gana Laura; si la suma de ambos es 7, gana María; y en cualquier otro caso hay empate.
Calcule las probabilidades de gane de cada una y diga cuál ene mayores chances de éxito.

42. ¿Cuántos cubos diferentes, con sus caras numeradas de 1 al 6 pueden ser fabricados, si la suma
de los números que se encuentran sobre cada par de lados opuestos debe ser 7?

43. ¿De cuántas maneras puede colorearse una cuadricula de 8 × 8 con los colores blanco y negro, de
tal manera que no existan dos columnas pintadas iguales?

44. ¿De cuántas manereas se pueden colocar las 32 piezas del ajedrez en el tablero sin que los reyes
se estén amenazando?

45. Dado el conjunto A = {1, 2, . . . , 10}, ¿cuántos cuadrados se pueden formar con los puntos de
A2 ?
Nota: los cuadrados pueden ser oblicuos también.

46. Se dibujan n cuerdas no concurrentes en una circunferencia (n ≥ 2). Estas cuerdas se cortan
en m puntos al interior de la circunferencia, los cuales subdividen a las cuerdas en r segmentos.
Determine r en función de n y m.

Nota: En la figura anterior, se han dibujado n = 4 cuerdas, que se cortan en m = 4 puntos, los
cuales subdividen a las cuerdas en r = 12 segmentos.

47. Sea X = {1, 2, 3, . . . , 100} y sea S = {(a, b, c) tales que a, b, c ∈ X, a < c y a < c}. ¿Cuál es
el cardinal de S?

19
Combinaciones y el número
1. Aprendizaje inicial de la Lengua es
combinatorio

20
2.1. MODELO DE CONJUNTOS
Dado un conjunto de n elementos (n ∈ N), ¿cuántos subconjuntos hay de k elementos (0 < k ≤ n)? A
este número le llamaremos el combinatorio de n escoger k.

Definición 2.1. Número Combinatorio: Dado un conjunto de cardinalidad n, el número de subconjuntos


de éste que enen cardinalidad k es el número combinatorio, denotado por

Cnk , (nk ) , o bien nCk

Esta claro que este número puede interpretarse como la can dad de formas de escoger k elementos de
entre los n disponibles.7

A pesar que de momento no podemos calcular este número, sí podemos hacer un uso habilidoso de él
a par r de su definición y de las propiedades que se derivan a par r de ésta.

Es importante mencionar que Cn0 significa la can dad de subconjuntos de cardinalidad cero que ene un
subconjunto de n elementos, y como el único subconjunto que cumple eso es ∅ entonces Cn0 = 1 para
cualquier natural n; también es fácil argumentar que Cnn = 1 para todo natural n.

EJEMPLO 2.1
Suponga que se ene el conjunto A = {a, e, i , o, u} y se busca la can dad de subconjuntos de A de 3
elementos:

{a, e, i }, {a, e, o}, {a, e, u}, {a, i , o}, {a, i, u}, {a, o, u}, {e, i , o}, {e, i, u}, {e, o, u}, {i, o, u}

En total resultaron 10 por lo que C53 = 10. Observe que para formar un subconjunto de 3 elementos
de un conjunto de 5 elementos, pues lo que se hace es escoger cuáles son los tres que conformarán el
subconjunto, y esto significa que al mismo empo se está no escogiendo a los restantes 2, por lo tanto,
por cada subconjunto de 3 elementos sí escogidos existe uno y sólo un subconjunto de no escogidos, y
viceversa; es decir, por el principio de biyección C53 = C52 . A con nuación se muestran los subconjuntos
de elementos no escogidos de la sucesión de subconjuntos anteriores:

{o, u}, {i , u}, {i , o}, {e, u}, {e, o}, {e, i }, {a, u}, {a, o}, {a, i }, {a, e}

Con este razonamiento y sin necesidad de construir todos los subconjuntos, si tenemos un conjunto de
cardinalidad 9 se tendría que: C90 = C99 , C91 = C98 , C92 = C97 , C93 = C96 , C94 = C95 ¿Observa cuál es el
patrón?

Teorema 2.1. Para todo n ≥ k se cumple la siguiente iden dad combinatoria

Cnk = Cnn−k
7
La primera notación u lizada puede tener inver do el orden de los parámetros dependiendo el autor que se consulte.

21
Demostración. Dado que en un conjunto de n elementos, por cada subconjunto de k elementos existe
uno y sólo un subconjunto de n − k elementos (una vez escogidos k, los n − k son los no escogidos),
y viceversa, entonces contar los subconjuntos de k elementos da el mismo resultado que contar los
subconjuntos de n − k elementos, y de allí se sigue el resultado.

EJEMPLO 2.2
Un combinatorio fácil de calcular es Cn1 , es decir, la can dad de subonjuntos unitarios de un conjunto
de n elementos, claramente Cn1 = n, y por la iden dad anterior también se cumplirá Cnn−1 = n.

EJEMPLO 2.3
Calculemos Cn2 .

Se trata por supuesto de elegir una pareja de elementos del conjunto que ene n elementos. Para
calcularlo podemos imaginar ordenados los elementos del conjunto dado. Consideremos primeramente
las parejas que incluyen el primer elemento; éste puede combinarse para formar la pareja con cualesquiera
de los (n−1) elementos restantes, en consecuencia hay (n−1) parejas que incluyen el primer elemento.
En seguida consideremos las parejas que no incluyen el primer elemento pero que incluyen el segundo
elemento que son en total (n − 2), ya que estas son las posibilidades de combinar el segundo elemento
con cada uno de los elementos restantes. Las parejas que no incluyen ni el primero, ni el segundo
elemento, pero sí el tercer elemento, son en número igual (n − 3), y así sucesivamente hasta llegar a
considerar las parejas que no incluyen a ninguno de los (n−2) primeros elementos, pero sí el penúl mo,
que únicamente puede ser combinado con el úl mo, dando así sólo 1 posibilidad. El análisis anterior nos
permite afirmar que el número Cn2 es igual a la suma de los números naturales desde el 1 hasta el (n−1),
cuyo resultado en representación compacta de sumatoria es:


n
n(n − 1)
Cn2 = i=
i=1
2

El método de análisis u lizado para calcular Cn2 , lleva implícito un método para generar las diferentes
alterna vas de formar los subconjuntos deseados, problema que con frecuencia es tan o más importante
que saber determinar el número de posibilidades. Sin embargo, cuando de determinar sólo el número
de alterna vas se trata, el método de conteo anterior no resulta tan cómodo y es preciso obtener otras
propiedades que muestran un poco más su lógica interna y que eventualmente simplifican los cálculos.

Por ejemplo, ya se demostró que la can dad de subconjuntos de un conjunto de n elementos es 2n ;


podemos recalcular este número dividiendo los subconjuntos en clases disjuntas: los subconjuntos de
cardinalidad 0, los subconjuntos de cardinalidad 1, los subconjuntos de cardinalidad 2, ..., los subconjuntos
de cardinalidad n −1, los subconjuntos de cardinalidad n. Dado que la can dad de subconjuntos de cada
una de estas clases me la da un número combinatorio, respec vamente Cn0 , Cn1 , Cn2 , . . . , Cnn−1 , Cnn , y por
el principio de la suma, se ene demostrado el siguiente teorema:

22
Teorema 2.2. Para todo n se cumple la siguiente iden dad combinatoria

n
2n = Cn0 + Cn1 + Cn2 + · · · + Cnn−1 + Cnn = Cnk
k=0

Este analisis realmente es fruc fero, podemos demostrar también este otro resultado

Teorema 2.3. Iden dad de Pascal: Para todo n y para todo k ≤ n se cumple la siguiente iden dad
combinatoria
Cnk = Cn−1
k−1 k
+ Cn−1

Demostración. Si deseamos calcular Cnk y consideramos un elemento en par cular, los dos posibles casos
(disjuntos) son: que se incluya o no éste elemento en los subconjuntos por formar. Si el elemento se
incluye, únicamente hace falta seleccionar (k − 1) elementos de los (n − 1) restantes para completar
k−1
los k, y el total de formas de hacer esto es Cn−1 ; si no se incluye hará falta seleccionar los k elementos
de los n − 1 elementos restantes, es decir, hay Cn−1
k
posibilidades en este caso. Luego, el teorema queda
demostrado por el principio de la suma.
Una de las técnicas básicas de conteo consiste en contar de dos maneras dis ntas un conjunto de objetos,
lo que permite establecer algunas iden dades. He aquí un ejemplo pico:

EJEMPLO 2.4
Supongamos que deseamos seleccionar una direc va de k personas en una asamblea de n personas y
en la direc va debe tenerse 1 presidente.

Si consideramos el problema de determinar el número de alterna vas que se enen para realizarlo,
éste podemos resolverlo en las dos formas siguientes: primero seleccionamos los k direc vos y en
seguida seleccionamos el presidente entre los k direc vos; o bien, primero seleccionamos el presidente
de entre los n asambleistas y en seguida seleccionamos los k − 1 direc vos restantes de entre los n − 1
asambleistas restantes. Al realizarlo de la primera de las formas el número de alterna vas es Cnk Ck1 ; en
k−1
la otra forma indicada, se ob ene Cn1 Cn−1 . Así, hemos logrado establecer las relaciones:

Cnk Ck1 = Cn1 Cn−1


k−1

kCnk = nCn−1
k−1

2.2. MODELO DE CAMINOS


Definición 2.2. Puntos Re culares: Llamaremos puntos re culares o puntos lá ces a puntos (a, b) en el
plano cartesiano tales que a, b ∈ Z.

Considere ahora los puntos re culares A(0, 0) y B(k, n − k) ubicados en el primer cuadrante del plano,
estos definen una re cula (o malla) tomando estos puntos como esquinas opuestas, y dicha re cula
formada por cuadraditos de lado 1. ¿Cuántos caminos sobre la re cula de longitud mínima hay de A a
23
B?

Observe que para que los caminos sean de longitud mínima, los movimientos permi dos sólo son hacia
la derecha → y hacia arriba ↑; así, en total, hay que avanzar k unidades hacia la derecha y n − k hacia
arriba. Entonces, el problema planteado lo puedo conver r en un problema de decisiones: me muevo
de punto re cular a punto re cular, hay que avanzar n unidades, avanzando unidad por unidad y a cada
paso decidiendo si es hacia la derecha o hacia arriba; pero k decisiones son obligatoriamente hacia la
derecha, y dependiendo cuáles de estas decisiones sean hacia la derecha se me definirá un camino (el
resto n − k decisiones forzosamente tendrán que ser hacia arriba), y viceversa. Ahora bien, del conjunto
de las n decisiones, la can dad de formas de escoger k para que sean movimientos hacia la derecha es
Cnk , por lo tanto, el número de caminos buscado es Cnk .

Teorema 2.4. El total de caminos sobre la re cula definida por A(0, 0) y B(k, n − k), y de longitud
mínima, es igual a Cnk

B(k, n − k)
...

.. . ..
. .. .

. ...
A(0, 0)

Un detalle interesante es que para llegar al punto B(k, n − k) existe únicamente dos formas: o se llega
por el punto re cular B1 (k −1, n −k), o bien por B2 (k, n −k −1); además, observe que estos casos son
excluyentes, porque un camino de A a B que pase por B1 y B2 tendrá que tener al menos un movimiento
hacia la izquierda o al menos un movimiento hacia abajo, por lo que ya no sería un camino de longitud
mínima.

Caso 1: El total de formas de llegar de A a B pasando por B1 (cumpliendo las exigencias, que sean
caminos de longitud mínima u lizando únicamente la re cula definida por A y B) es, por el principio de
la mul plicación, el total de formas de llegar de A a B1 por el total de formas de llegar de B1 a B. Dado
que para llegar de A a B1 se toman en total n − 1 decisiones, de las cuales k − 1 son hacia la derecha, el
k−1
total de caminos de longitud mínima de A a B1 es Cn−1 ; además, como el total de formas de llegar de
B1 a B u lizando caminos de longitud mínima es 1, se ene que el total de formas de llegar de A a B
pasando por B1 es Cn−1k−1
· 1 = Cn−1
k−1
.

Caso 2: Este se resuelve de forma muy similar al caso 1, el total de formas de llegar a A a B pasando
por B2 es igual al producto de la can dad de formas de llegar de A a B2 por la can dad de formas de
24
llegar de B2 a B. Para llegar de A a B2 , en total se toman n − 1 decisiones, de las cuales k son hacia la
k
derecha, por lo que el total de caminos de A a B2 es Cn−1 . De aquí se concluye fácilmente que el total
k
de caminos del caso 2 es Cn−1 .

Finalmente, por el principio de la suma, dado que estos casos son excluyentes, el total de caminos de A
a B es igual al total de caminos de A a B pasando por B1 más el total de caminos de A a B pasando por
B2 , es decir

Cnk = Cn−1
k−1 k
+ Cn−1
Se obtuvo de nuevo la Iden dad de Pascal (2.3). Esta iden dad proporciona un algoritmo (no muy
eficiente) para calcular el número combinatorio de forma recursiva, ya que expresa el número combinatorio
de subíndice n en términos de combinatorios de subíndices n − 1, que a su vez podrían escribirse en
términos de combinatorios de subíndies n − 2, y así sucesivamente hasta llegar a combinatorios de fácil
cálculo. A con nuación mostramos una tabla

k 0 1 2 3 4 5 6 7 8 9
n
0 1
1 1 1
2 1 2 1
3 1 3 3 1
4 1 4 6 4 1
5 1 5 10 10 5 1
6 1 6 15 20 15 6 1
7 1 7 21 35 35 21 7 1
8 1 8 28 56 70 56 28 8 1
9 1 9 36 84 126 126 84 36 9 1

Este modelo del número combinatorio es independiente del modelo de conjuntos, observe que sin
mucho esfuerzo pueden hacerse las deducciones Cn0 = 1 = Cnn , Cn1 = n y Cnk = Cnn−k . ¡Inténtelo!
De mayor reto es por ejemplo demostrar por este método el teorema (2.2); este lo podemos analizar
de la siguiente forma: considere la siguiente cuadrícula, el total de caminos de longitud n que parten de
A(0, 0) formados sólo con movimientos → y ↑ puede contarse de dos formas dis ntas, la primera por
el principio de la mul plicación, como en cada uno de los n movimientos se enen las dos opciones, en
total son 2n caminos; la segunda es dividir estos caminos en casos excluyentes, los caminos que terminan
en B0 (0, n), los que terminan en B1 (1, n − 1), y así sucesivamente, hasta contar los que terminan en
Bn (n, 0)

25
B0 (0, n)

B1 (1, n − 1)

B2 (2, n − 2)

.
A(0, 0) Bn (n, 0)

Estos conteos devuelven Cn0 , Cn1 , . . . , Cnn , respec vamente, y de allí que

2n = Cn0 + Cn1 · · · + Cnn

2.3. CADENAS DE CEROS Y UNOS


Muchos problemas combinatorios, mediante la biyección adecuada, pueden transformarse a problemas
de cadenas de ceros y unos, el número combinatario por sí mismo es un ejemplo de ello.

Teorema 2.5. La can dad de cadenas de k ceros y n − k unos es Cnk .

Demostración. Hasta el momento se ha dicho que Cnk pude interpretarse 1) Como la can dad de subconjuntos
de k elementos de un conjunto de cardinalidad n, o lo que es lo mismo, es la can dad de formas de
escoger k objetos de entre n objetos diferentes; y 2) representa la can dad de caminos de longitud
mínima (sobre la malla) que van del punto A(0, 0) al punto B(k, n − k).

Ahora bien, ambas interpretaciones del número combinatorio pueden asociarse a un tercer problema:
¿Cuántas cadenas de ceros y unos se pueden formar con k ceros y n − k unos?

Una cadena de k ceros y n − k unos puede interpretarse de la siguiente forma: Dado el conjunto A =
{x1 , x2 , . . . , xn }, cada elemento puede o no ser escogido para conformar a un subconjunto; si denotamos
por 0 a ``sí se escoge'' y por 1 a ``no se escoge'', una cadena de k ceros y n − k unos representa la sí
escogitación de k elementos de A y la no escogitación de los restantes n − k, es decir, hace referencia la
conformación de un subconjunto de k elementos de A; además, la posición de los ceros hace referencia
a qué elemento de A se escogió; por ejemplo

011
| {z· · · 0}
n

representa que sí se escoge a x1 , que no se escogen x2 y x3 ,..., que sí escoge xn . El orden entre los ceros
y unos puede cambiar, pero la can dad de ceros debe ser k.

Con los caminos, la biyección es aún más evidente: a un movimiento a la derecha (avanza una unidad
horizontalmente) se le asocia un 0, en total son k de estos movimientos, y a un movimiento hacia arriba
26
se le asocia un 1, que son en total n − k; por tanto, la can dad de cadenas con k ceros y n − k unos es
igual a la de caminos de longitud mínima desde A(0, 0) a B(k, n − k), que son en total Cnk .

Finalmente, se puede dar una aproximación al problema de las cadenas de k ceros y n − k unos de
forma directa: puedo interpretar esto como que si se enen a disposición n espacios vacíos en los que
se colocará o bien un 0 o bien un 1; además, se ene prefijada la can dad de ceros y unos que deben
colocarse en esos n espacios, en total son exactamente k ceros y obviamente los restantes n − k deben
ser unos. Está claro que si se colocan de alguna manera los k ceros, las posiciones que deben ocupar
entonces los unos están ya fijadas y no hay nada más que hacer, entonces, me basta conocer de cuántas
formas puedo colocar los ceros y luego, las posiciones de los unos quedan determinadas; pero este
conteo es familiar: ¿De cuántas formas puede escoger k espacios vacíos (para colocar los ceros) de los
n espacios vacíos disponibles?... pues de Cnk formas.

2.4. TRIÁNGULO DE PASCAL


La siguiente pirámide de números es conocida como Triángulo de Pascal

1 1

1 2 1

1 3 3 1

1 4 6 4 1

1 5 10 10 5 1

1 6 15 20 15 6 1

1 7 21 35 35 21 7 1

1 8 28 56 70 56 28 8 1

1 9 36 84 126 126 84 36 9 1
..
.
Cada fila está formada por la anterior, cada número es igual a la suma de los dos números que se ubican
ligeramente a la izquierda y a la derecha de la fila anterior, y el número de la primera fila es 1; en caso
de que sólo tenga uno de esos números, se considera igual a cero al que no aparece en la pirámide.
Teorema 2.6. Los números del triángulo de Pascal son números combinatorios.
Demostración. Dado que el primer número del triángulo es 1, la forma de construcción y la iden dad
de Pascal (2.3), el teorema queda demostrado.
27
2.5. BINOMIO DE NEWTON
Considere la expresión
n v eces
z }| {
(x + y ) = (x + y )(x + y ) · · · (x + y )
n

Si se efectúan todos los productos aparecerán los términos x n , x n−1 y , x n−2 y 2 , . . . , xy n−1 , y n con sus
respec vos coeficientes; entonces la pregunta es ¿cuál es la relación que determina el coeficiente de
cada término x k y n−k ? ¿Esta relación depende de n, de k o de ambos? Analicemos algunos casos par culares:

(x + y )0 = 1
(x + y )1 = x +y
(x + y )2 = x 2 + 2xy + y 2
(x + y )3 = x 3 + 3x 2 y + 3xy 3 + y 3
(x + y )4 = x 4 + 4x 3 y + 6x 2 y 2 + 4xy 3 + y 4
(x + y )5 = x 5 + 5x 4 y + 10x 3 y 2 + 10x 2 y 3 + 5xy 4 + y 5
..
.

Si observamos los coeficientes, aparece la siguiente secuencia, ¿la reconoce?

1
1 1
1 2 1
1 3 3 1
1 4 6 4 1
1 5 10 10 5 1
..
.
¡Claro!, vuelven a aparecer los combinatorios, se ob ene el Triángulo de Pascal. Ahora la cues ón es
jus ficar por qué los coeficientes del desarrollo (x + y )n son números combinatorios. Lo primero que
hay que hacer es entender cómo se ob ene un coeficiente en par cuar al desarrollar (x + y )n . Veamos
un ejemplo par cular:

EJEMPLO 2.5
Considere (x + y )4 = (x + y )(x + y )(x + y )(x + y ), para desarrollarlo, de cada paréntesis debemos
escoger la x o la y , y luego se mul plican esos cuatro números, al recorrer todas las posibles escogitaciones
se ob enen todos los términos, que luego, se suman términos semejantes. Veamos esto en acción:

Se escribirán las escogitaciones así: xxy y representa escoger del primer paréntesis la x, del segundo
paréntesis la x, del tercer paréntesis la y , del cuarto paréntesis la y . Entonces, las posibles escogitaciones
son, por el principio de la mul plicación, 25 = 32 (dos opciones por cada paréntesis); listadas una a una
se ob ene

28
xxxx = x4 xy xx = x 3y y xxx = x 3y y y xx = x 2y 2
xxxy = x 3y xy xy = x 2y 2 y xxy = x 2y 2 y y xy = xy 3
xxy x = x 3y xy y x = x 2y 2 y xy x = x 2y 2 yyyx = xy 3
xxy y = x 2y 2 xy y y = xy 3 y xy y = xy 3 yyyy = y4
Luego, el coeficiente representa la can dad de veces que aparece el mismo término, por ejemplo, observe
que el término x 2 y 2 aparece 6 veces, entonces en el desarrollo de (x + y )4 uno de los sumandos es
6x 2 y 2 . Centremos nuestra atención en este término, hay que buscar un método que permita calcular
ese 6 de forma sistemá ca, quizás esto dé una pista para abordar el caso general:

Para que aparezca el término x 2 y 2 , se necesita claramente un par de x y un par de y , pero una vez que
se escoge el par de paréntesis de los cuáles se selecciona la x, está claro que de los otros dos paréntesis
deberá escogerse la y ; así, el total de forma de escogerse 2 paréntesis de los 4 es C42 = 6.

Exactamente los mismo sucede en el caso general, dado (x + y )n , las veces que aparecerá el término
x k y n−k es igual a la can dad de formas de escoger k paréntesis de los n (en cada uno de estos paréntesis
se seleccionará la x, en los n − k se seleccionará la y ), y esto es Cnk . Así, se ob ene la siguiente iden dad
algebraica llamada el Binomio de Newton:

Teorema 2.7. Binomio de Newton: El desarrollo del binomio (x + y )n es


n
(x + y ) =n
Cnn x n + Cnn−1 x n−1 y + Cnn−2 x n−2 y 2 + ··· + Cn1 xy n−1 + Cn0 y n = Cnk x k y n−k
k=0

Esta iden dad es muy ú l para encontrar y demostrar iden dades combinatorias, observe por ejemplo

EJEMPLO 2.6
Si x = y = 1:

2n = (1 + 1)n = Cnn + Cnn−1 + Cnn−2 + · · · + Cn1 + Cn0

Nos encontramos de nuevo con (2.2 ), que ya se derivó a par r de los subconjuntos de un conjunto de
cardinalidad n, u lizando caminos y ahora con el binomio de Newton. Otro ejemplo

EJEMPLO 2.7
Suponga que x = 1, y = −1, se ene entonces

(1 − 1)n = Cnn − Cnn−1 + Cnn−2 − Cnn−3 + · · · + (−1)n−k Cnk + · · · + (−1)n Cn0


∑ ∑
Cnk = Cnk
k par k impar

Esta es una relación cuya deducción por los métodos anteriores es tremandamente di cil; con la iden dad
de Newton se puede ir muy lejos, incluso, es posible evaluar en x o en y valores complejos.
29
Lectura Complementaria: Eder Alexander Jacobo. Material de Autoformación Docente para Matemá ca
9° grado. Lección 7. Número Combinatorio. MINED

2.6. PROBLEMAS
1. ¿De cuántas formas pueden 4 niñas y 4 niños ser dividos en dos equipos de 4 si cada equipo debe
tener al menos a una niña?
2. ¿De cuántas formas pueden 5 niñas y 3 niños ser dividos en dos equipos de 4 si cada equipo debe
tener al menos a un niño?
3. En una rifa par cipan 100 personas, regalándose 3 televisores idén cos. Quién diseñó la rifo pensó
que sacar un cket, luego el segundo y finalmente el tercero era injusto, por lo que se sacan al
mismo empo los 3 ckets. ¿Cuál es la probabilidad que una persona que compró un solo cket
gane?
4. Una asamblea de 14 personas desea elegir entre sus miembros un presidente, un vicepresidente
y un secretario de actas. ¿De cuántas formas puede realizarse la elección?
5. Ocho promotores deben visitar 4 comercios. Para ello forman 4 parejas, debiendo cada una de
ellas visitar un establecimiento. ¿De cuántas formas pueden distribuirse el trabajo?
6. Adrián ene nueve pares dis ntos de calce nes, un día se levantó tarde para su clase de las 9:00AM
y en la prisa tomó aleatoreamente ocho calce nes sin mirarlos. ¿Cuál es la probabilidad de que
entre los calce nes tomados hayan exactamente dos pares correctos?
7. Se dispone de una colección de 30 pelotas divididas en 5 tamaños dis ntos y 6 colores diferentes,
de tal manera que en cada tamaño hay pelotas de todos los colores. ¿Cuántas colecciones de 4
pelotas enen exactamente 2 pares de pelotas del mismo tamaño (que no sean las 4 del mismo
tamaño)?
8. En un curso de Combinatoria hay 30 estudiantes, de los cuales 19 son mujeres. Para organizar
una feria de ciencias, se pide que de este curso se presenten tres exposiciones, realizadas por
parejas de estudiantes, tal que cada pareja está intengrada por estudiantes de dis nto género, y
un estudiante no puede estar en dos o más parejas. ¿De cuántas formas puede hacerse esto?
9. Se desea elegir una direc va, hay diez candidatos (cinco mujeres y cinco hombres) para los cargos
de presidente, vicepresidente, secretario, tesorero y vocal. ¿De cuántas formas pueden elegirse
los cargos? ¿De cuántas formas es posible hacerlo si una mujer será la presidenta? ¿De cuántas
maneras es posible si el tesorero ya está definido que será Juan?
10. Una persona ene seis amigos. Cada noche, durante cinco días, invita a cenar a un grupo de tres de
ellos, de modo que el mismo grupo no sea invitado dos veces. ¿De cuántas formas puede hacerlo?
11. Dados los conjuntos A = {1, 2, . . . , 6} y B = {1, 2, . . . , 10}, ¿cuántas funciones estrictamente
crecientes8 f : A → B pueden definirse? ¿cuántas además cumplen que f (4) = 7?
8
Se dice que f es estrictamente creciente si a < b ⇒ f (a) < f (b).
30
12. Se dice que una mano de dominó (compuesta por 7 fichas de dominó) ene ``falla'' si alguno de
los números entre el 0 y el 6 no aparece en la mano. Determine el número de manos de dominó
que no enen falla.

13. En cada subconjunto de 7 elementos del conjunto {1, 2, . . . , 10} se escoje el elemento mayor.
¿Cuál es la suma de todos los elementos mayores?

14. Diez puntos están marcados en el plano, no habiendo tres colineales. ¿Cuántos triángulos pueden
formarse?

15. ¿Cuántos cuadriláteros (convexos o no convexos) pueden formarse con los vér ces de un n−ágono
regular?

16. Dados seis puntos sobre una circunferencia, se trazan todas las cuerdas que estos puntos definen.
¿Cuál es la probabilidad de que al escoger aleatoreamente cuatro de estas cuerdas se forme un
cuadrilátero convexo?

17. De cierto número de rectas coplanares se sabe que no hay tres de ellas que concurran en el mismo
punto y no hay ninguna pareja de rectas paralelas. Esas rectas producen 45 puntos al cortarse. ¿De
cuántas rectas estamos hablando?

18. ¿Cuántos puntos de intersección producen 8 rectas coplanares, sabiendo que dos de ellas son
paralelas y no hay tres concurrentes?

19. Se enen nueve puntos en un plano. Cuatro de ellos están alineados y los restantes están dispuestos
de forma que no hay nunca 3 alineados. ¿Cuántos triángulos pueden formarse que tengan sus
vér ces sobre esos 9 puntos? ¿Cuántas rectas dis ntas determinan esos puntos?

20. En una fábrica hay varios centros de almacenamiento, cada uno de los cuales está unido a los
demás por una cinta transportadora. Calcula el número de centros de la fábrica si se sabe que el
número de cintas transportadoras es 66.

21. Dibuja una circunferencia y marca sobre la misma diez puntos. Uniendo parejas de esos puntos
¿Cuántos pentágonos convexos dis ntos se podrían formar?

22. Dado el conjunto de dígitos {1, 3, 6, 7, 9}, determine el número de maneras de formar números
de 4 cifras tales que sean múl plos de 3.

23. ¿Cuántos números de cuatro cifras cumplen la propiedad de que el producto de dichas cifras es
un cuadrado perfecto?

24. Un cuadrado de lado 6 es dividido en 36 cuadrados de lado 1. Los puntos A y B son puntos medios
de un par de lados opuestos del cuadrado. A cada lado de la línea AB seis cuadrados unitarios son
tomados aleatoriamente y coloreados de azul. Si el cuadrado grande es doblado sobre recta AB,
calcule la probabilidad de qué exactamente un par de cuadrados azules coincidan.

25. ¿De cuántas maneras puede distribuirse 3n objetos dis ntos en tres cajas dis ntas de modo que
cada caja tenga el mismo número de objetos?

31
26. Calcule cuántas cadenas de ceros y unos de longitud 7:

a) con enen exactamente 3 unos y 4 ceros.


b) con enen 4 unos consecu vos.
c) con enen como máximo 3 unos.
d) no con enen 4 o más unos consecu vos.
k−1
27. Demuestre la iden dad de Pascal Cnk = Cn−1
k
+ Cn−1 u lizando dis ntos modelos combinatorios.
k−1 k−2
28. Demuestre que Cnk = C20 Cn−2
k
+ C21 Cn−2 + C22 Cn−2 .
k−1
29. Demuestre Cnk = Cn−1 k−1
+ Cn−2 + · · · + Ck−1
k−1
es válido para todo k ≤ n.

30. Demuestre que



2n−1 k
C2n
=0
k=0
(−2)k

31. Demuestre

n
3r Cnr = 4n
r =0

32. Hallar el número de caminos crecientes que empiezan en (0, 0) y terminan en alguno de los puntos
indicados.

33. Demuestre por diversos métodos la iden dad de Vandermonde


1 r −1
0 r
Cm Cn + Cm Cn + · · · + Cm
r
Cn0 = Cm+n
r

34. En la cuadrícula de abajo

B(10, 10)

X(6, 4)

.
A(0, 0)

a) ¿De cuántas maneras podemos ir de la casilla A a la casilla B con movimientos siempre hacia
la derecha o bien una casilla hacia arriba?
32
b) ¿De cuántas maneras se puede hacer esto si debemos pasar además por la casilla X?
c) ¿Cuántos caminos llevan de A a B sin pasar por ningún punto de úl ma línea ver cal (salvo B
naturalmente)
d) ¿De cuántas maneras se llega de A a B pasando por un solo punto de la segunda línea horizontal?

35. Demuestre por diversos métodos la siguiente iden dad



n
( )2
n
C2n = Cnj
j=0

36. Dentro de un paralelepípedo rectangular de alambre A de dimensiones 5 × 4 × 6 de colocan


alambre dividiendo a A en cubos de lado 1. ¿Cuántos caminos diferentes de longitud mínima hay
desde el vér ce inferior izquierdo de la cara anterior de A hasta el vér ce superior derecho de la
cara posterior de A?

37. Suponga que ene una armazón cúbica de alambre de l × m × n (la armazón está formada por
cubitos de alambre lado 1). Si una hormiga está ubicada en una esquina y quiere llegar caminando
por los alambres a la esquina más alejada con el mínimo recorrido ¿de cuántas maneras puede
hacerlo? ¿y de cuántas formas podría hacerlo si la hormiga camina sobre una armazón 4−dimensional
de l × m × n × k? ¿puede generalizar para más dimensiones?

38. Dado un polígono conexo tal que no ene tres diagonales concurrentes, ¿cuántos puntos de intersección
al interior del polígono forman las diagonales?

39. Demuestre la iden dad

Cpq Cr0 + Cp−1 Cr +1 + · · · + Cp−q


q−1 1 0
Crq+q = Cp+r
q
+1

p−r
40. Demuestre que Cnp Cpr = Cnr Cn−r siempre que n ≥ p ≥ r .

41. Calcule
n (
∑ )
1
k+ Cnk
k=1
k

42. Demuestre que



n
1 ( 2n )
j n 2
C2n = C4n + (C2n )
j=0
2

43. Demuestre las iden dades combinatorias de Chu Shih Chieh

Crr + Crr+1 + · · · + Cnr = Cn+1


r +1

Cr0 + Cr1+1 + · · · + Crk+k = Crk+k+1

33
44. Demuestre la iden dad
k 2 = 2Ck2 + Ck1
y a par r de ella demuestre que
( )
1 1
1 + 2 + ··· + n = n n +
2 2 2
(n + 1)
3 2

45. Demostrar que para todo natural n se cumple


( )
n+1 2n + 1 n
C2(n+1) =2 C2n
n+1

46. Resuelva las siguiente ecuaciones combinatorias:

a)
2 2 2
Cm−1 + Cm + Cm+1 = 19
b)
x−1 x
7C2x−2 = 2C2x

c)
x2
Cx0 + Cx1 · Cx2 = +2
2
d)
n3 − 6n2 + 20
Cn3 − Cn2 =
6
e)
Cn4 = 13Cn2

f)
1 2 3
C2n + C2n + C2n = 387n

g)
n−5 n−7
Cn−1 = Cn−3
523 524
47. Calcule el valor de C525 + C525
( )67
20x 3 y 2
48. En la expansión de + , determinar:
y5 x
a) El vigésimo término
b) El coeficiente del término cuarenta y cinco
c) El coeficiente de x 23 , si es posible. Si no, argumente el por qué no existe.
d) ¿Cuántos términos ene la expansión?

49. Encuentre el coeficiente de x 5 en la expansión de (1 + x + x 2 )8 + (1 + x + x 2 )9 .


34
( )5
1 6 7
50. En la expansión de x + 3 + x + x encontrar el coeficiente de x 4 .
x
51. Encuentre el coeficiente del término a7 b4 ce 2 en el desarrollo de (a + b + c + d + e)14 .
12!
52. ¿Cuál es la suma de todos los números de la forma si a, b, c varían sobre todos los enteros
a!b!c!
no nega vos que cumplen que a + b + c = 12?
1 25
53. Dada la expresión (7x 2 + 2x ) , encuentre la can dad de términos de su expansión, la suma de
los coeficientes de su expansión, el coeficinte de x 1 3 si es posible, y encuentre el i -ésimo término.
k−1 k+1 k k k+1
54. Demuestre la iden dad del hexágono: Cn−1 Cn Cn+1 = Cn−1 Cnk−1 Cn+1 . El nombre viene del
hecho que los coeficientes binomiales implicados forman una figura hexagonal alrededor de Cnk
en el triángulo de Pascal.

55. Encuentre el coeficiente de x n y de x n+r (con 1 ≤ r ≤ n) en la expansión de

(1 + x)2n + x(1 + x)2n−1 + x 2 (1 + x)2n−2 + · · · + x n (1 + x)n

56. ¿Cuántos paralelogramos quedan determinados cuando un grupo de 6 rectas paralelas es intersecado
por otro grupo de 6 rectas paralelas?

57. Un tablero de 5 × 5 es dividido en 25 cuadrados unitarios, de estos dos son pintados de azul y el
resto pintado de blanco, diremos que dos coloraciones son iguales si una puede ser obtenida de
la otra al rotar tablero. ¿Cuántas coloraciones dis ntas existen?

58. Un polígono convexo de n lados es tal que no hay punto común alguno para cualesquiera tres de
sus diagonales. Determine el número de triángulos que se forman de manera tal que dos de sus
vér ces sean vér ces del polígono y el tercero sea una interseccción de dos diagonales.

59. Una rana se ubica en el tercer escalón de unas gradas, la rana se mueve un escalón por salto.
¿Cuántas formas existen para que la rana llegue por primera vez al octavo escalón en su noveno
salto?

60. ¿De cuántas formas pueden ordenarse los enteros del 1 al n bajo la siguiente condición: excepto
por el primer entero de la izquierda, todo entero debe diferir por 1 de algún entero que esté más
la izquierda que este?

35
Permutaciones y arreglos
2. Aprendizaje de la lectura inicial

36
3.1. PERMUTACIONES
Sea A un conjunto finito. Ordenar elementos de A es darle a cada elemento del conjunto una posición
determinada, es decir definir qué elemento ocupa la primera posición, el que ocupa la segunda posición y
así sucesivamente. Dos ordenamientos de elementos de A se dirá que son idén cos si todos los elementos
en ambos ordenamientos se encuentran en la misma posición; en consecuencia dos ordenamientos
serán diferentes si difieren en la posición en la que se encuentra alguno de los elementos. Hay diversas
formas de ordenar los elementos de un conjunto y en este apartado nos ocuparemos de contar el total
de alterna vas de ordenamiento de los elementos de un conjunto de cardinalidad n.

EJEMPLO 3.1
¿De cuántas formas pueden ordenarse los elementos del conjunto A = {a, b, c}?

Los elementos del conjunto A = {a, b, c} se pueden ordenar de las siguientes seis formas: abc, acb, bac, bca, cab
y cba. Observe que para contar los posibles ordenamientos nos basta definir el elemento que ocupará la
primera posición, el que ocupará la segunda y el de la tercera posición, lo que es equivalente a contar las
ternas (x, y , z) de elementos de A con la condición que los elementos de la terna sean todos diferentes.
En este caso, la primera posición puede ser ocupada por uno cualesquiera de los elementos de A, es decir
contamos en este caso 3 posibilidades; para la segunda posición ya sólo disponemos de 2 posibilidades
y para la posición tercera sólo hay 1 posibilidad.

En general cuando el conjunto A ene n elementos, el número total de formas de ordenarlos es n! puesto
que la primera posición puede ser ocupada por uno cualesquiera de los n elementos de A; la segunda
posición por cualquier elemento que no sea el colocado en la primera posición, por lo que las alterna vas
se reducen a (n − 1); para la tercera posición se enen (n − 2) alterna vas y así sucesivamente dejando
a la úl ma posición con una única alterna va, y por el principio de la mul plicación las alterna vas de
ordenar los elementos de A serán n(n−1)(n−2) · · · (2)(1) = n!. A cada ordenamiento de los elementos
de A se le denomina una permutación, y al total de permutacione se denota por Pn . Tenemos entonces
el resultado siguiente:
Teorema 3.1. El total de permutaciones de n elementos, denotado por Pn es

Pn = n!

3.2. PERMUTACIONES CIRCULARES


Supongamos que debemos colocar a n personas alrededor de una mesa circular en la que se dispone de n
posiciones numeradas para su ubicación y deseamos determinar el total de alterna vas de ordenamiento.
Si dos ordenamientos dados los consideramos diferentes cuando las personas ocupan posiciones numeradas
diferentes, entonces el número de posibles ordenamientos será igual a un ordenamiento en una fila, es
decir, Pn ; a estas les llamaremos permutaciones lineales. Sin embargo puede ser que sólo nos interese la
posición rela va que guardan entre sí las personas y no nos interese el número de la posición que ocupan,
a estas permutaciones les llamaremos permutaciones circulares; si este es el caso hay n permutaciones
37
lineales que dejarían a las personas con la misma permutación circular, en efecto, la rotación de las
personas pasando por las n posiciones numeradas dejan a las personas en la misma posición rela va. En
consecuencia hay n permutaciones lineales por cada permutación circular; como el total de permutaciones
lineales es Pn , el total de permutaciones circulares, el cual denotamos por Cn , será
Teorema 3.2. El total de permutaciones circulares de n elementos, denotado por Cn es
Pn
Cn = = (n − 1)!
n
Hay otra manera de abordar esta mismo problema: Supongamos que una persona X se sienta primero,
es indis nto qué asiento escoge dado que lo importante es cómo se ubiquen las restantes n−1 personas,
numeramos 1 al asiento que escogió X y a par r de este se numeran los restantes asientos (en orden
horario o an horario, no importa); ahora las n − 1 personas restantes pueden permutarse linealmente
de Pn−1 formas, y una vez que han decidido una permutación lineal en par cular, toman los asientos
2, 3, . . . , n en el orden prefijado, eso da nuevamente Cn = Pn−1 = (n − 1)!

Ahora considere un problema parecido pero que entre líneas lleva una restricción más, suponga que se
enen 5 objetos dis ntos y se quieren elaborar todos los collares posibles u lizando estos cinco objetos,
¿cuántos hay? ¡Inténtelo!

3.3. PERMUTACIONES CON REPETICIÓN


Suponga ahora un problema dis nto: se quieren permutar n objetos, pero no todos son dis ntos, hay
dos clases de objetos, hay k1 objetos idén cos de la primera clase y k2 objetos idén cos de la segunda
clase (obviamente k1 + k2 = n), ¿cuántas permutaciones hay?

EJEMPLO 3.2
¿Cuántas permutaciones se pueden construir con 2 bolas negras idén cas y 2 bolas blancas también
idén cas?

Si las 4 bolas fuera todas dis ntas, el total de permutaciones (lineales) sería 4!; enlistemos todas las
posibilidades, se denotará por n1 y n2 a las bolas negras, y por b1 y b2 las blancas:

n1 n2 b1 b2 n1 b1 n2 b2 n1 b1 b2 n2 b1 b2 n1 n2 b1 n1 b2 n2 b1 n1 n2 b2
n1 n2 b2 b1 n1 b2 n2 b1 n1 b2 b1 n2 b2 b1 n1 n2 b2 n1 b1 n2 b2 n1 n2 b1
n2 n1 b1 b2 n2 b1 n1 b2 n2 b1 b2 n1 b1 b2 n2 n1 b1 n2 b2 n1 b1 n2 n1 b2
n2 n1 b2 b1 n2 b2 n1 b1 n2 b2 b1 n1 b2 b1 n2 n1 b2 n2 b1 n1 b2 n2 n1 b1
Ahora, observe que, dado que las dos bolas negras son iguales entre sí, y las dos bolas blancas tambien
son iguales entre sí, cada columna, realmente representa a la misma permutación, si les quitamos los
subíndices quedaría

nnbb nbnb nbbn bbnn bnbn bnnb


nnbb nbnb nbbn bbnn bnbn bnnb
nnbb nbnb nbbn bbnn bnbn bnnb
nnbb nbnb nbbn bbnn bnbn bnnb
38
Entonces, el total de permutaciones dis ntas bajo estas condiciones no es 4! sino que sólo 6.

¿Cómo podríamos calcular este 6 sin necesidad de hacer una por una todas las permutaciones?, la idea
central está allí, se trata de, en primer lugar, suponer que todos los objetos son dis ntos, entonces,
el total de permutaciones ya se sabe como calcularlo, es 4!; pero, dado que no todos los objetos son
dis ntos, hay que ver cuántas veces aparece una verdadera permutación en el conteo anterior; tome
por ejemplo bnnb, cuando se suponen todos dis ntos, las letras b se pueden permutar de 2! formas
mientras que las letras n se pueden permutar de otras 2! formas, por lo que bnnb aparecerá 2!2! = 4
veces cuando consideremos dis ntos a los objetos (coincide con la tabla, ¿cierto?). Entonces 4! es el
cuádruple de las permutaciones que buscamos, es decir, que las permutaciones con objetos repe dos
4!
son en total = 6.
2!2!

Resolvamos ahora el caso general: se enen k1 objetos de una clase y k2 objetos de otra clase (los objetos
de una misma clase son idén cos entre sí), y denotamos por P (k1 , k2 ) al total de permutaciones de estos
objetos. Tomando n = k1 + k2 , si todos los objetos fueran dis ntos, el total de permutaciones sería Pn ,
pero en este conteo, cada permutación de las buscadas aparece repe da k1 !k2 ! veces, por lo que

Teorema 3.3. El total de permutaciones con repe ción de k1 objetos de un po y k2 objetos de otro po,
denotado por P (k1 , k2 ) y tomando n = k1 + k2 es

n!
P (k1 , k2 ) =
k1 !k2 !
Este argumento se puede generalizar:

Teorema 3.4. Dados k1 objetos idén cos de una clase 1, luego otros k2 objetos idén cos de una clase 2,
..., kr objetos idén cos de una clase r, y tomando n = k1 + k2 + · · · + kr , el total de permutaciones es

n!
P (k1 , k2 , . . . , kr ) =
k1 !k2 ! · · · kr !
Por otra parte, ya antes se había trabajado un problema muy similar a este, el de las cadenas de longitud
n de ceros y unos, todos los ceros son iguales entre sí, todos los unos también, y se está interesado en
la can dad de cadenas dis ntas que se pueden formar con k ceros y n − k unos, es decir, todas las
permutaciones con k objetos de idén cos de una clase y n − k de la otra; además, ya se demostró que
el total de estas cadenas es Cnk , entonces

Teorema 3.5. El número combinatorio puede interpretarse como permutaciones con elementos repe dos,
y se relacionan así
n!
Cnk = P (k, n − k) =
k!(n − k)!
Nota: En la relación anterior, observe que pueden darse los casos k = 0 (sólo hay unos) y k = n (sólo
hay ceros), y según los modelos de número combinatorio, Cn0 = 1 = Cnn , por lo que para que la relación
anterior siga siendo válida en estos casos, se define 0! = 1.

39
3.4. ARREGLOS
Regresamos a ordenar objetos dis ntos, pero considerando que la can dad de espacios es inferior o
igual a la can dad de objetos dis ntos que se ordenan; es decir, ¿cuántas configuraciones de longitud k
puede formarse si se enen n objetos dis ntos (k ≤ n)? A estas configuraciones les llamaremos arreglos,
y al total de arreglos de longitud k dados n objetos dis ntos se le denotará por Akn .9

Este problema es una aplicación directa del principio de la mu plicación, y sin duda podemos argumentar
el caso general sin inconveniente alguno: se enen k espacios, en el primero puede ubicarse a cualquiera
de los n objetos, hay etonces n opciones; en el segundo espacio puede ubicarse a cualquiera de los n
objetos exceptuando el que se colocó en la primera posición, por lo que hay n − 1 opciones; en el
tercer espacio puede colocarse a cualquiera de los n objetos exceptuando el que se colocó en la primera
posición y el que se colocó en la segunda posición, por lo que hay n−2 opciones; este argumento se sigue
así sucesivamente, hasta que en el espacio k puede ubicarse a cualquiera de los n objetos, exceptuando
los que se colocaron en las anteriores k − 1 posiciones, por lo que hay n − (k − 1) opciones. Así

Teorema 3.6. El total de arreglos de longitud k dados n objetos dis ntos (k ≤ n) es

Akn = n(n − 1)(n − 2) · · · (n − (k − 1))

Pero esta expresión puede escribirse de forma más compacta mul plicanndo y dividiendo por (n − k)!

n(n − 1)(n − 2) · · · (n − (k − 1))(n − k)! n!


Akn = =
(n − k)! (n − k)!
En par cular si n = k se ob ene
Ann = Pn
Observe que Cnk y Akn cuenta estructuras similares, la diferencia radica que cuando se calcula el combinatorio,
únicamente interesa escoger k objetos de los n disponibles (el orden de los objetos no importa), en
cambio, cuando se calcula la can dad de arreglos, luego de escogerlos se construyen las k! permutaciones
(el orden sí importa), entonces

Teorema 3.7. Dados n objetos dis ntos, la can dad de arreglos y de combinaciones de longitud k (k ≤ n)
se relacionan por
Akn = k!Cnk

Lectura Complementaria: Eder Alexander Jacobo. Material de Autoformación Docente para Matemá ca
9◦ grado. Lección 6. Permutaciones. MINED

3.5. PROBLEMAS
1. Para confeccionar un examen, se dispone de 3 problemas de Geometría, 4 de Combinatoria y 2
de Álgebra. ¿De cuántas maneras pueden ordenarse los problemas si los que corresponde a un
mismo tema deben aparecer en forma consecu va?
9
La notación más usual para este número es Pnk , y a estas se les llama también permutaciones, pero dado que permutar
significa únicamente cambiar de orden, reservaremos esa palabra exclusivamente para el caso en el que hay igual número de
objetos que de espacios.
40
2. El equipo de Anita y el de Chepito juegan futbol con la regla adicional siguiente: gana quién primero
ob ene cuatro goles y no hay empates. ¿De cuántas formas puede ganar el equipo de Anita?

3. ¿De cuántas formas es posible escoger 4 cartas de dis nto manjar de una baraja de 52 cartas?

4. Un bebé recién nacido puede tener 1, 2 ó 3 nombres. ¿De cuántas formas puede llamarse si se
puede escoger de 300 nombres disponibles?

5. En una mesa redonda de 5 asientos se sientan 7 personas. ¿De cuántas formas pueden hacerlo si
la persona 1 es enemiga de la persona 2 y si se sienta una no se sienta la otra?

6. ¿De cuántas formas pueden entregarse 6 cartas urgentes si se enen a disposición 3 couriers
dis ntos?

7. ¿Cuántas permutaciones hay que no tengan I juntas de la palabra PARANGUATIRIMICUARO?

8. A una conferencia han sido invitadas como exponentes 5 personas: A, B, C, D y E. ¿De cuántas
formas se pueden ordenar las exposiciones si B no debe preceder a A? ¿Cuántas formas dis ntas
serían si B debe hablar inmediatamente después que A?

9. ¿De cuántas formas pueden sentarse 5 mujeres y 5 hombres en una mesa redonda de tal forma
que las personas vecinas de cada quien son de género dis nto?

10. ¿Cuál es la can dad de configuraciones dis ntas que pueden generar los m semáforos ubicados
sobre una calle principal?

11. Una madre ene 2 manzanas y 3 peras, y le da a su hija una fruta cada día (de lunes a viernes), ¿de
cuántas formas puede hacerlo? ¿De cuántas formas podría hacerlo si ene además 4 naranjas?

12. Una profesora distribuye 5 naranjas dis ntas entre sus 8 estudiantes de tal forma que cada estudiante
recibe a lo sumo una naranja, ¿de cuántas formas puede hacerlo? ¿Cuántas formas serían si ya no
se restringe la can dad que puede recibir cada estudiante?

13. Un club depor vo de 30 miembros quiere hacer 4 equipos de 4 personas cada uno para que
par cipen en un rally, ¿de cuántas formas puede hacerlo?

14. ¿De cuántas formas pueden distribuirse m + n + p objetos dis ntos en 3 cajas tal que cada una
tenga m, n y p objetos respec vamente?

15. Determine el número de permutaciones de la palabra TONACATEPEQUE.

16. Dada una baraja de 4 manjares y 52 cartas (diamantes, corazones, espadas y tréboles, 13 cartas de
cada manjar), ¿de cuántas maneras es posible escoger 4 cartas de dis nto manjar tal que el valor
de la carta de diamantes sea igual al valor de la de corazones, y el valor de la carta de espadas sea
igual al valor de la de tréboles?

17. Con una baraja igual a la del ejercicio anterior, determine la probabilidad de obtener en la primera
mano de un juego de póker: una escalera de color, un póker, una escalera, color, una casa llena,
un trío, dos pares.
41
18. Si se juega con 6 dados, dos negros, dos verdes y dos rojos, ¿cuál es la probabilidad de obtener
una escalera tal que los dados del mismo color tengan números consecu vos? (considere al 6 y al
1 como consecu vos también)

19. En El Salvador, los números telefónicos se forman con 8 cifras, siendo 2 el primero de ellas, y la
segunda 2, 3, 4, 5 ó 6; en Guatemala en cambio son sólo 7 cifras pero con las mismas restricciones
para sus primeras dos cifras. ¿Cuántas comunicaciones pueden establecerse entre los habitantes
de ambos países?

20. ¿Cuántos bloques coloreados diferentes, de forma cúbica fija, puede hacerse si hay seis colores
disponibles y cada bloque debe tener un color diferente en cada una de sus seis caras?

21. La familia Pérez ha comprado una mesa circular nueva con seis sillas, justa para toda la familia. Para
esta familia, no hay preferencias con respecto a cuál silla ocupar, sólo les interesa saber quienes
enen a la par. ¿De cuántas formas pueden sentarse si Juanita y Pedro están peleados y no quieren
sentarse juntos? ¿y si la mamá Pérez quiere sentarse a la par de Anita?

22. Hallar la can dad de números que se obtengan como permutaciones del número 111122256 que
sean divisibles por 12.

23. Seis matrimonios se reúnen a cenar en una mesa circular. ¿De cuántas formas pueden ubicarse, si
cada hombre debe estar flanqueado por dos mujeres y los miembros de cada pareja deben estar
juntos?

24. Cuatro bailarines y cuatro bailarinas interpretan una danza que consiste en formar una ronda
tomados de la mano. ¿De cuántas formas pueden ubicarse si en la figura deben aparecer alternadamente
hombres y mujeres?

25. Sean k, n ∈ N. Demuestre que el número de formas de sentar kn personas alrededor de k mesas
dis ntas de modo que hay n personas en cada mesa es

(kn)!
nk

42
Extensiones del número
4.
combinatorio

43
4.1. SEPARADORES
Considere el siguiente problema:

EJEMPLO 4.1
Ana quiere comprar 10 dulces para regalárselos a sus primitos; en la enda hay dulces de tres sabores,
menta, fresa y limón, ¿de cuántas formas puede escogerlos?

Si llamamos m a la can dad de dulces de menta, f la can dad de dulces de fresa y l la can dad de dulces
de limón, debe cumplirse que m + f + l = 10, y obviamente cada uno de estos números es mayor o
igual a cero. Como siempre, lo mejor es analizar algunos casos par culares, por ejemplo m = 0, entonces
f + l = 10, y las parejas solución son

(f , l) = (0, 10), (1, 9), (2, 8), (3, 7), (4, 6), (5, 5), (6, 4), (7, 3), (8, 2), (9, 1), (10, 0)

en total hay 11 posibilidades. Si analizamos el resto de casos (todos disjuntos) m = 1, 2, . . . , 10, se


obtendrán respec vamente 10, 9, . . . , 1 posibilidades, por lo que la can dad de formas que Ana puede
hacer la compra es 11 + 10 + · · · + 1 = 66.

Este método resolvió el problema y las cuentas no fueron largas ni di ciles, sin embargo no nos da idea
de como abordar un problema con más variables o con números más grandes. Por ejemplo, si los sabores
disponibles fueran 8 y la can dad de dulces que Ana comprará es 100, el problema se vuelve muchísimo
más di cil y el método anterior no funcionará bien.

Si se observa, este problema es dis nto a todos los estudiados hasta el momento, y básicamente se trata
de buscar combinaciones de objetos, pero no todos los objetos son dis ntos (para el caso, los dulces de
fresa los consideramos todos iguales, los de menta también y los de limón también). Hay una forma muy
ingeniosa de resolver este problema, y por lo importante de la técnica, suele dedicársele una sección.

Haremos lo siguiente: los 10 dulces los vamos a interpretar como 10 objetos iguales, 10 bolas por
ejemplo, y para dis nguir cuáles son de cada sabor, incluiremos 2 ``separadores''; luego, estos 12 objetos
se permutan, m es la can dad de bolas que quedan a la izquierda del primer separador, f es la can dad
de bolas que quedan entre los separadores y l es la can dad de bolas que quedan a la derecha del
segundo separador. Además, la can dad de permutaciones con repe ción con 10 bolas y 2 separadores
12!
es P (10, 2) = = 66.
10!2!



⃝⃝ ⃝ ⃝ ⃝⃝ ⃝ ⃝ ⃝⃝
| {z} | {z } | {z }
m f l

El problema general se resuelve de la misma forma:

44
Teorema 4.1. Dada una colección de objetos clasificados en k pos de objetos (los objetos del mismo po
son iguales entre sí, y dis ntos de cualquier objeto de otro po), el total de formas de escoger n objetos
es
(n + k − 1)!
P (n, k − 1) =
n!(k − 1)!

⃝ |⃝⃝| || ⃝ ⃝ ⃝ |⃝ ⃝ · · · ⃝| ⃝ ⃝

Demostración. Se considera en principio que los n objetos son todos iguales, y para distribuir los objetos
entre las posibles k clases, se agregan k − 1 separadores. El total de configuraciones es igual a las
permutaciones con repe ción P (n, k − 1).
Observe que la respuesta puede verse como un combinatorio también: en total, se enen n + k − 1
espacios y se escogen los n (o bien los k − 1) en los que se ubican las bolas (o bien los separadores), por
lo que la can dad de configuraciones buscadas es

n k−1
Cn+k−1 o bien Cn+k−1
Finalmente, una versión muy u lizada de separadores es la siguiente:

Teorema 4.2. El total de soluciones enteras no nega vas de la ecuación x1 + x2 + · · · + xk = n es Cn+k−1


n

La relación con el problema anterior es evidente, porque xi representa la can dad de objetos del po i ; el
par cular, el problema de Ana es equivalente a resolver la ecuación x1 +x2 +x3 = 10, con xi ∈ Z+ 0 . Note
que hay configuraciones que enen cero bolas de algún po, esto, en el esquema de los separadores, se
da cuando los separadores están juntos, o cuando un separador está a la izquierda de todas las bolas o
a la derecha de todas las bolas.

Otro detalle importante que comentar es que a veces se busca configuraciones que tengan al menos uno
de cada po, es decir, xi ≥ 1. En tal caso los separadores se ubican únicamente en los n − 1 espacios
entre las n bolas, a lo sumo un separador por espacio; así, el total de configuraciones con esta nueva
k−1
restricción es Cn−1 .

4.2. MULTICOMBINATORIO
()
Dados n objetos dis ntos, el total de formas de escoger p de ellos es pn ; esto, como ya se estudió en
reiteradas ocasiones, puede interpretarse con modelo de cajas: en la caja 1 se introducen los p objetos sí
escogidos, y en la caja 2 los n − p objetos no escogidos, así, este problema es equivalente a: ¿De cuántas
formas es posible distribuir n objetos dis ntos en dos cajas, tal que en la primera caja siempre hayan
p objetos? Lo interesante de este planteamiento es que se puede extender fácilmente a más número
de cajas; al número que obtendremos en ese caso se le llama mul combinatorio, y no es más que una
extensión y generalización del número combinatorio.

45
Definición 4.1. Mul combinatorio: Al total de formas de distribuir n objetos dis ntos en k cajas, tal que
en la primera caja siempre hayan x1 objetos, en la segunda caja x2 objetos, ..., en la k−ésima caja xk
objetos, con x1 +x2 +· · ·+xk = n, se le llama mul combinatorio de n escoger x1 , x2 , . . . , xk , y se denota
( )
n
x1 , x 2 , . . . , x k

Es interesante que este problema puede resolverse sin mayores dificultades a par r del número combinatorio
estándar, lo cual se enuncia en el siguiente teorema

Teorema 4.3. El mul combinatorio de n escoger x1 , x2 , . . . , xk es:


( ) k (
∏ )
n n − (x1 + x2 + · · · + xs−1 )
=
x1 , x 2 , . . . , x k s=1
xs

Demostración.
( n ) De los n objetos escogemos los x1 que se colocarán en la primera caja, lo cual se puede
hacer de x1 formas; luego, escogemos los x2 objetos de entre los n − x1 objetos restantes, que se
( 1)
colocarán en la segunda caja, lo cual es posible de n−xx2 formas, y así sucesivamente, hasta que para la
(xk ) de entre los n − (x1 + x2 + · · · + xk−1 ) = xk objetos, lo cual se
caja k−ésima se escogerán xk objetos
podrá hacer sólo de una forma, xk . Por lo tanto, el total de posibilidades es
( )( )( ) ( )
n n − x1 n − (x1 + x2 ) n − (x1 + x2 + · · · + xk−1 )
···
x1 x2 x3 xk

Este número también ene modelos equivalentes por caminos y por relaciones algebraicas. El vínculo
del mul combinatorio con expresiones algebraicas viene de desarrollar los mul (nomios,
) por ejemplo
n a b c n
(x + y + z) , el coeficiente de x y z (con a + b + c = n) es el mul combinatorio a,b,c ; ¡demuéstrelo!

Además, es importante que iden fique el nexo entre separadores y el mul combinatorio, son problemas
aparentemente iguales, pero son muy dis ntos; observe por ejemplo que el total de términos del desarrollo
del trinomio (x + y + z)n es igual a la can dad de términos x a y b z c , y esto es igual a la can dad de
2
soluciones de a + b + c = n, lo cual, por separadores es Cn+2 .

4.3. PROBLEMAS
1. Determine el número de formas que pueden ordenarse en un estante 4 libros dis ntos de Combinatoria,
5 libros dis ntos de Geometría, 3 libros dis ntos de Álgebra y 8 libros dis ntos de Cálculo, si los
de Geometría deben estar siempre antes que los de Álgebra.

2. ¿En cuántas de las permutaciones del número 23814425 aparecen los dígitos impares en forma
creciente de izquierda a derecha?

3. Se han encargado 20 pupusas de entre los siguientes pos: revueltas, de queso, de chicharrón, de
frijol con queso, de queso con loroco y de ayote. ¿De cuántas formas puede hacer la compra?
46
a) Si se ene que llevar al menos 7 de queso.
b) Si se ene que llevar a lo sumo 2 de chicharrón y 10 de ayote.
c) Si se ene que llevar al menos 3 de cada clase.

4. Encuentre el número de secuencias no-decrecientes de largo 10

a1 ≤ a2 ≤ a3 ≤ · · · ≤ a10

donde ai ∈ {1, 2, 3, . . . , 100}

5. ?`Cuántas 11-combinaciones pueden formarse de las letras x, y , z si todas las letras deben aparecer
al menos dos veces y a lo sumo 5?

6. ?`Cuántas cadenas existen de 10 dígitos ternarios (0, 1 ó 2) que contengan exactamente dos 0,
tres 1, y cinco 2?

7. ?`Cuántas soluciones existen para la desigualdad

x1 + x2 + x3 ≤ 11

donde x1 , x2 y x3 son enteros no nega vos?

8. ?`De cuántas formas pueden ordenarse n ceros y k − 1 unos si no hay dos 1 consecu vos?

9. Se ran 12 dados idén cos al aire, ¿cuál es la probabilidad de que uno de los números 1, 2, . . . , 6
no aparezca?

10. ¿De cuántas formas puede distribuirse 20 bolas iguales en 6 cajas, de tal forma que en la primera
caja hay al menos 4 bolas y en la ul ma caja no más de 5?

11. Existen 5 formas de expresar el número 4 como suma de dos enteros no nega vos tomando en
cuenta el orden: 4 = 0+4 = 1+3 = 2+2 = 3+1 = 4+0. Dados los naturales r y n, determine:

a) El número de formas de expresar 200 en r sumandos.


b) El número de formas de expresar n en 200 sumandos.
c) El número de formas de expresar n en r sumandos tales que todos sean mayores o iguales que
5.

12. José comprará 20 galletas de dis ntos sabores: fresa, chocolate, vainilla y limón. ¿De cuántas
formas puede hacer la compra si ene que llevar un número de galletas de vainilla que sea 4
veces el número de galletas de limón?

13. Determine el número de soluciones enteras no nega vas de 3x1 + 5x2 + x3 + x4 + x5 = 20.

14. Determine el número de términos de la expansión de (x1 + x2 + x3 + · · · + xn−1 )n−1 .

15. Determine el número de soluciones enteras no nega vas de r x1 + x2 + x3 + · · · + xn = kr .

16. En la expansión de (a + b + c + d)48


47
17. Encuentre el número de enteros posi vos x tales que x ≤ 9999999 y la suma de sus dígitos sea
igual a 31.

18. Si Lorena ra un dado cinco veces, ¿cuál es la probabilidad de que la sumas de sus cinco radas
sea 20?

19. ¿Cuántas soluciones hay, entre 1 y 9 inclusive, de la ecuación x1 + x2 + x3 + x4 = 26?

20. Encuentre el número de soluciones enteras posi vas de la ecuación:

(x1 + x2 + x3 )(y1 + y2 + y3 + y4 ) = 77

a) Encontrar el coeficiente de a8 b10 c 15 d 15 .


b) Determinar el número de términos de la expansión.
6
21. Determine el coeficiente de x 5 en la expansión de (1 + x + x 2 + · · · + x 1000 ) .

4.4. PRINCIPIO DE INCLUSIÓN - EXCLUSIÓN


El principio, puede ser extendido extendido de forma natural al caso de la reunión de n conjuntos. La
forma que adopta el principio en este caso es:

Teorema 4.4. Principio de Inclusión - Exclusión: Dados n conjuntos A1 , A2 , . . . , An , el cardinal de la unión


está dado por

n
∪ ∑ ∑ ∑

Ai = |Ai | − |Ai ∩ Aj | + |Ai ∩ Aj ∩ Ak | + · · · + (−1)n+1 |A1 ∩ A2 ∩ · · · ∩ An |

i=1 i i<j i<j<k

La fórmula pretende asegurar que todos los elementos son contados una y sólo una vez, lo que puede
ser verificado después de un momento de reflexión.

4.5. DESÓRDENES

EJEMPLO 4.2
Si consideramos el orden de los números naturales 1, 2, 3, 4, en la permutación 3142 ningún elemento
está en su posición natural. A una permutación con tal propiedad le denominaremos un desorden o un
desarreglo. ¿De los 24 posibles ordenamientos de tales números, cuántos desórdenes existen?

Denotemos por D4 tal número de desórdenes; por F1 el número de las permutaciones que dejan fijo
un elemento en su posición; por F2 las que dejan en su posición dos de los 4 elementos, por F3 las que
dejan en su puesto tres de los 4 elementos; de manera completamente similar definiremos F4 . Así, por
el principio de inclusión-exclusión (4.4) tenemos el siguiente resultado:

D4 = 4! − F1 + F2 − F3 + F4
48
Ahora bien, F1 se descompone en los que dejan fijo el 1 en su posición, que son 6, los que dejan fijo el
2, que son otras 6; hay 6 que dejan el 3 en su posición y 6 que dejan fijo el 4; así, F1 es 24. De las que
dejan fijos dos de los cuatro elementos están los que dejan fijos el 1 y 2, los que dejan fijos el 1 y 3, el 1
y 4; los que dejan fijos el 2 y 3, el 2 y 4; finalmente tenemos los que dejan fijos el 3 y el 4. Como en cada
uno de estos caso, que son seis, el número de permutaciones es 2, resulta que F2 es 12. El caso de que
dejen fijos tres elementos con ene los casos siguientes: que dejen fijos 1, 2, 3; 1, 2, 4; 1, 3, 4 y el caso
que dejen fijos 2, 3, 4; en total son cuatro casos y cada uno de ellos ene una permutación por lo que
F3 es igual a 4. Finalmente, F4 con ene una única permutación, es decir F4 es 1. En resumen tenemos:

D4 = 24 − 24 + 12 − 4 + 1 = 9
Hay en consecuencia 9 desórdenes en las permutaciones de orden 4.

Esta claro que esta relación se puede generalizar.

Teorema 4.5. El total de desórdenes de orden n, denotado por Dn , es


∑n ( )
k n
Dn = (−1) (n − k)!
k=0
k
Demostración. El total de permutaciones es Pn = n!. Tomando la misma notación del ejemplo anterior,
Fk denota aquellas permutaciones que enen a k (al menos) de sus elementos en la posición que les
corresponde; así, el total de de permutaciones de Fk lo contamos primero escogiendo los k que quedarán
en la posición que les corresponde, lo cuál es posible hacerlo de Cnk formas, y luego permutando los
restantes n − k objetos, lo cual es posible hacerlo de Pn−k = (n − k)! formas, y por el principio de la
mul plicación, Fk = Cnk (n − k)!. Luego, por el principio de inclusión-exclusión:
Dn = n! − F1 + F2 − F3 + · · · + (−1)k Fk + · · · + (−1)n Fn
= (−1)0 Cn0 (n − 0)! + (−1)1 Cn1 (n − 1)! + (−1)2 Cn2 (n − 2)! + · · · + (−1)n Cnn (n − n)!
∑n ( )
k n
= (−1) (n − k)!
k=0
k

Observe que esa expresión se puede manipular algebraicamente, y reescribirse como


n
(−1)k
Dn = n!
k=0
k!

4.6. RECURRENCIA

EJEMPLO 4.3
La Torre de Hanoi:
Se dispone de n discos de diferentes medidas y de tres clavijas en donde éstos pueden ser colocados.
Inicialmente todos los discos están colocados en una de las clavijas y ordenados de abajo hacia arriba de
mayor a menor. El problema consiste en trasladarlos a otra de las clavijas siguiendo las reglas siguientes:
49
) Los discos se mueven uno por uno.

) En ningún caso puede colocarse un disco sobre un disco de radio menor.

Obsérvese que para cumplir con la condición segunda resulta indispensable disponer de tres clavijas.
El problema consiste en determinar el número mínimo de movimientos requeridos para pasar todos los
discos de una clavija a otra.

Sea mn el número mínimo de movimientos para pasar n discos de una clavija a otra. Es obvio que para
poder mover el disco que se encuentra en el fondo, debemos pasar de una clavija a otra los n − 1 discos
menores a otra clavija, lo que requiere mn−1 como número mínimo de movimientos. Hecho lo anterior,
debemos pasar el disco mayor a la tercera clavija, lo que exige un movimiento, y finalmente pasar los
n−1 discos menores a esta úl ma clavija, lo que por definición requiere nuevamente mn−1 movimientos
como mínimo. Así en total se requieren como mínimo mn = 2mn−1 + 1 movimientos. Por otra parte,
cuando se ene sólo 1 disco, obviamente, el número mínimo de movimientos es m1 = 1; es decir:
{
mn = 2mn−1 + 1
m1 = 1

Información suficiente para calcular el valor de mn para los diferentes valores de n.

EJEMPLO 4.4
La Sucesión de Fibonacci:

Suponemos que cada mes la hembra de una pareja de conejos pare una pareja de conejos (de diferente
sexo). Dos meses más tarde la hembra de la nueva pareja pare una nueva pareja. Determinar el número
de parejas de conejos al finalizar el primer año, si se supone que al inicio sólo se dispone de una pareja
de conejos en edad reproduc va y que en el período en mención no hay defunciones.

Por ejemplo, al final del primer mes tendremos dos parejas, al final del segundo mes se tendrán tres
parejas. Denotemos por Fn el número de parejas al final del enésimo mes. Este número de parejas debe
ser igual al número de parejas en el mes n − 1, que es igual a Fn−1 , más los recién nacidos, cuyos padres
únicamente pueden ser las parejas existentes en el mes n − 2. Así, resulta la relación:

 Fn = Fn−1 + Fn−2
F = 1
 0
F1 = 2
De nuevo, con estas relaciones podemos calcular el valor de Fn para los diferentes valores de n. Con
ella se logra la conocida como sucesión de Fibonacci: 1, 2, 3, 5, 8, 13, 21, 34, 55, . . .. El problema de
50
determinar el número de parejas de conejos al final del primer año es simplemente el de calcular F12 ,
que es igual a 377.

EJEMPLO 4.5
El Modelo de Reproducción de Conejos y las Cadenas de ceros y unos que no enen dos unos seguidos.

Veremos como este problema del número de parejas de conejos al final del enésimo mes puede ponerse
en correspondencia con el problema de las cadenas de ceros y unos de longitud n que no enen dos unos
seguidos.

Suponga una cadena de ceros y unos ene la propiedad de no tener dos unos consecu vos. Cada 1 en la
cadena representará el nacimiento de una pareja; cada pareja de conejos, se iden ficará con el úl mo
1 en la cadena; los demás unos de la cadena que aparecen antes, representarán el árbol genealógico
de la pareja que representa la cadena. La cadena de sólo ceros estará asociada a la pareja original. La
restricción de que las cadenas no tengan dos 1 consecu vos es justamente debido a que en el modelo
cada pareja sólo puede comenzar a procrear al finalizar el segundo mes.
Por ejemplo, la cadena 00101001 es el documento de iden dad de la pareja de conejos que nació al
finalizar el octavo mes, cuyos padres nacieron en el quinto mes; sus abuelos nacieron al finalizar el tercer
mes y sus bisabuelos la pareja de conejos original.

Es evidente que esta correspondencia es biunívoca; a cada cadena de ceros y unos con la propiedad de
no tener dos unos seguidos, le corresponde una sola pareja de conejos y a cada pareja de conejos, le
corresponde una sola cadena.

Podemos ahora, apoyados en la correspondencia anterior, proponernos contar el número de parejas


de conejos, mediante el conteo de las cadenas de ceros y unos con la propiedad de no tener dos unos
seguidos. Sólo con el propósito de ilustrar nos restringiremos al caso de cadenas de longitud 6, el caso
general puede ser analizado de forma completamente análoga. El problema es así el de contar las cadenas
de longitud 6 de ceros y unos con la propiedad de no tener dos unos seguidos.

Obsérvese primero que para que no hayan dos unos seguidos se requiere que el número de unos sea
menor o igual a tres, de lo contrario no se dispondrá de suficientes ceros para separarlos. Podemos
entonces clasificar nuestras cadenas según el número de unos que posea: Cadenas que no enen unos;
los 6 son ceros y los espacios disponibles para colocar los unos, son 7. Se ene entonces C70 en este caso.
Cadenas que enen exactamente un uno. Obviamente la cadena debe tener 5 ceros, lo que permite
disponer de 6 espacios para colocar el uno, en este caso el número de cadenas debe ser C61 . He aquí la
lista de todas estas cadenas: 100000, 010000, 001000, 000100, 000010, 000001. Cadenas que enen
exactamente dos unos. En este caso deben haber cuatro ceros, lo que permite disponer de 5 espacios
para colocar los 2 unos. El total deben ser C52 cadenas, que son los siguientes: 101000, 100100, 100010,
100001, 010100, 010010, 010001, 001010, 001001, 000101. Cadenas que enen tres unos. Por supuesto
la cadena ene tres ceros y los espacios disponibles para colocar los tres unos, son 4; por lo tanto
tendremos C43 cadenas; estas son: 101010, 101001, 100101, 010101.

51
El total de cadenas con la propiedad será entonces la suma: 1+6+10+4 = 21, número que corresponde
a F6 en la sucesión de Fibonacci.

4.7. PRINCIPIO DE CASILLAS


El principio de casillas es posiblemente uno de los teoremas más evidentes, y cualquiera podría pensar
que no ene u lidad alguna una aseveración tan evidente, sin embargo, en los problemas que se plantean
en este capítulo se puede apreciar lo poco evidente que resultan muchos problemas cuya clave radica
en el principio de casillas.

EJEMPLO 4.6
Una recta no puede cortar internamente a los tres lados de un triángulo simultáneamente.

En este caso hay que crear tanto las cajas como los objetos; digamos que se traza la recta L,10 la cual
genera dos semiplanos, éstos serán las cajas; por otra parte, los vér ces del triángulo serán los objetos,
que son en tres en total. Por el principio de casillas, hay un semiplano que ene al menos dos vér ces,
por lo tanto, la recta L no corta al lado definido por esos vér ces.

4.8. PROBLEMAS
1. ¿Cuántos enteros entre 1 y 1000, incluyéndolos, no son divisibles entre 2, 3 ó 5?

2. ¿Cuántos números naturales menores o iguales que 10000 son múl plos de 4, 5 ó 7? ¿Cuántos
son múl plos de 4, 10 ó 14?

3. En cierta escuela hay 100 alumnos. De ellos, 50 saben inglés, 30 saben alemán y 30 saben francés.
Además, 10 saben inglés y francés, 14 saben francés y alemán, 11 saben inglés y alemán, y 6 saben
los tres idiomas. Determinar cuántos alumnos no saben ninguno de los tres idiomas.

4. Un grupo de 102 estudiantes se examinan en Matemá cas, Sociales y Lenguaje. De entre ellos, 92
pasaron Matemá cas, 75 Sociales y 63 Lenguaje, 65 pasaron Matemá cas y Sociales, 54 Matemá cas
y Lenguaje y 48 Sociales y Lenguaje. ¿Cuántos estudiantes pasaron las tres materias?

5. ¿Cuántos números del 1 al 1000000 no son ni cuadrados perfectos, ni cubos perfectos, ni potencias
cuartas perfectas?

6. En un grupo de 100 indios hay 40 que hablan hindi, 40 que hablan bengalí y 20 que hablan penjabi.
Hay 20 que hablan hindi y bengalí, y 5 que hablan hindi y penjabi. Hay 31 que hablan al menos dos
de las tres lenguas y 33 que no hablan ninguna de ellas. ¿Cuántos hablan las tres lenguas?

7. ¿Cuántos números menores que 400 enen la propiedad de no tener divisor alguno en el conjunto
{6, 10, 15}?
10
Como se busca cortar internamente a los lados del triángulo, suponemos que L no coincide con ninguno de los lados ni
pasa por algún vér ce.
52
8. ¿Cuántos números naturales menores que 1000 enen la propiedad de ser divisibles por 12, pero
no divisibles ni por 5, ni por 7?

9. De todas las combinaciones posibles de 5 elementos del conjunto {1, 2, . . . , 10} ¿Cuántas no
incluyen ni el número 8, ni el número 9?

10. Determine el número de desórdenes posibles del conjunto {1, 2, 3, 4, 5}.

11. Si se asume el orden natural en el conjunto {1, 2, 3, 4, 5}, ¿cuántas permutaciones dejan fijos en
su posición exactamente a dos de los cinco números?

12. Considere los números naturales menores o iguales a 100 en su descomposición en factores primos.
¿En cuántos de ellos no hay un factor primo repe do?

13. Un año es bisiesto cuando:

) Es múl plo de 4 pero no de 100, o


) Es múl plo de 400.

Por ejemplo, 1600 y 1924 fueron años bisiestos, mientras que 2200 no lo será. Encuentre el
número total de años bisiestos entre los años 1000 y 3000.

14. ¿Cuántas permutaciones de 1234 son tales que el 1 no está en la primera posición, el 2 no está en
la segunda posición, el 3 no está en la tercera posición, y el 4 no está en la cuarta posición?

15. En una oficina hay 10 empleados. Cada uno es especialista en una labor dis nta a la de los demás.
Para no aburrirse, les gusta intercambiar sus puestos; sin embargo, el buen funcionamiento de la
oficina exige que en cada momento haya exactamente 4 empleados trabajando en su especialidad.
¿Cuántas distribuciones de los puestos se pueden hacer bajo estas condiciones?

16. Se sabe que la clave de acceso de una comutadora es una permutación de CLAVE2654. Sin embargo,
como el dueño sufre de dislexia, programó en su compuradora la siguiente regla: Para conseguir
acceso, es suficiente introducir 5 caracteres en la posición correcta, los caracteres restantes pueden
estar en la posición correcta o no. ¿De cuántas formas puede acceder a la computadora?

17. Ana quiere comprar 15 dulces, los sabores disponibles son: coco, vainilla, fresa y canela. Como a
su mamá le gustan los dulces de coco, piensa llevar al menos 5 de este sabor; además, nunca ha
probado los dulces de canela, por lo que llevará a lo sumo 4 de este sabor. ¿De cuántas formas
puede Ana comprar los dulces?

18. Para comprar en una carnicería cada cliente toma un número (los números van saliendo ordenados
1, 2, . . . , n), pero el carnicero estaba de mal humor un día y decidió atender a las n personas que
esperaban su turno de una manera muy extraña. ¿De cuántas formas puede atenderlos si los que
enen número impar serán atendidos en su turno, mientras que los que enen número par no?

19. En un grupo de 102 estudiantes se examinan en Matemá cas, Sociales y Lenguajes. De entre
ellos, 92 pasaron Matemá cas, 75 Sociales y 63 Lenguaje, 65 pasaron Matemá cas y Sociales, 54
Matemá cas y Lenguaje, y 48 Sociales y Lenguaje. ¿Cuántos estudiantes pasaron las tres materias?
53
20. Para rendir un examen, 4 alumnos se sientan en una fila de 10 asientos. ¿De cuántas maneras
pueden ubicarse, si no puede haber alumnos sentados con guamente?

21. Siete libros diferentes, tres de Historia, dos de Matemá ca y dos de Química, son colocados en el
estante de una biblioteca. ¿En cuántas de las posibles formas de ubicarlos no aparecerán todos los
libros de una misma materia?

22. ¿De cuántas formas se puede colocar tres x, tres y y tres z de modo que no aparezcan la misma
letra tres veces consecu vas?

23. En cierto ecosistema hay 18 especies de animales. Cada especie depredadora caza 2 especies
diferentes. A su vez cada especie no depredadora es perseguida por 3 especies diferentes. Además,
se sabe que toda especie es perseguida o depredadora, y ninguna de las dos cosas a la vez. ¿Cuántas
especies depredadoras hay?

24. Consideremos una cuadricula de n × n ¿cuántos cuadrados tales que apoyen alguno de sus lados
en el borde inferior o el borde izquierdo de de la cuadricula?

25. Consideremos una cuadricula de 8 × 10 casillas. ¿Cuántos rectángulos pueden marcarse en ella
que tengan por lo menos uno de sus lados apoyado en algún borde?

26. ¿Cuántas soluciones enteras ene la ecuación x1 + x2 + x3 = 28 si las variables están sujetas a la
condición: 3 ≤ x1 ≤ 9, 0 ≤ x2 ≤ 8, 7 ≤ x3 ≤ 17?

27. ¿Cuántas soluciones enteras ene la ecuación x1 + x2 + x3 + x4 = 20 si las variables están sujetas
a la condición x1 ≤ 6, x2 ≤ 7, x3 ≤ 8, x4 ≤ 9?

28. Hallar el número de soluciones enteras de la ecuación x1 + x2 + x3 + x4 + x5 + x6 = 20 que


sa sfacen las condiciones: 1 ≤ x1 ≤ 6, 1 ≤ x2 ≤ 7, 3 ≤ x3 ≤ 9, 4 ≤ x4 ≤ 11.

29. ¿De cuántas formas se pueden distribuir 10 premios dis ntos entre 4 estudiantes de modo que
exactamente dos estudiantes no reciban ninguno? ¿De cuántas formas puede hacerse esto de
modo que al menos dos estudiantes no reciban premio?

30. Demuestre mediante la definición de coeficiente mul nomial que:


( ) m (
∑ )
n n−1
= (1)
n 1 , n2 , . . . , n m i=1
n1 , . . . , ni − 1, ni + 1, . . . , nm

31. Demuestre que


∑( n
)
= mn
n1 , n 2 , · · · , n m
donde el sumatorio contempla todas las soluciones enteras no nega vas de n1 + n2 + · · · nm = n

32. Sea k, n, r ∈ N. Muestre que el número de soluciones enteras de la ecuación

x1 + x2 + x3 + · · · + xn = r

54
tal que 0 ≤ xi ≤ k para cada i = 1, 2, . . . , n está dado por
∑ n ( )( )
i n r − (k + 1)i + n − 1
(−1)
i=0
i n−1

33. Determinar el número de regiones determinadas por n rectas en el plano.


34. Determine el número de cadenas de ceros y unos que enen la propiedad de no tener dos unos
consecu vos.
35. Supongamos que se desea cubrir un rectángulo de dimensión n × 2 con rectángulos de 2 × 1. ¿De
cuántas formas es posible hacerlo?
36. ¿De cuántas formas es posible descomponer un número natural como suma sólo de los naturales
1 y 2?
37. Llamamos triangulación de un polígono a su descomposición en triángulos cuyos lados son ya sean
diagonales o lados del polígono, de forma tal que el polígono queda completamente cubierto, sin
superposiciones.
a) Demuestre que, independiente de la triangulación, el número de triángulos u lizados en la
descomposición es el mismo e igual a n − 2.
b) Demuestre que el número de diagonales u lizadas en cualquier triangulación es siempre n −3.
c) Determinar el número de posibles triangulaciones de un polígono de n lados.
38. Considere un triángulo equilátero cuyo lado ene longitud n. Se triángula trazando rectas paralelas
a los lados que cortan segmentos de 1 a los lados del triángulo; se forman triángulos equiláteros
de diferentes longitudes de lado. Determine el número total de tales triángulos.
39. Se dispone de un número ilimitado de monedas de las denominaciones de 1,5,10 y 25 centavos
de dólar. ¿De cuántas formas se puede formar un total de 30 centavos? Determine una relación de
recurrencia que permita calcular el número de posibilidades de cambiar una can dad cualquiera
de dinero.
40. Sean C un conjunto con 2n números reales, n ≥ 1 y an el número de comparaciones que deben
efectuarse entre los elementos de C para determinar el máximo y el mínimo de C. Encontrar una
relación de recurrencia para calcular an y resolverla.
41. Tenemos cinco puntos con coordenadas enteras en el plano cartesiano. Probar que si sacamos los
puntos medios para cada par de estos puntos, exis rá un punto medio cuyas coordenadas serán
también enteras.
42. Una prueba de concurso posee diez preguntas de selección múl ple, con cinco alterna vas cada
una. ¿Cuál es el número mínimo de candidatos que deberían hacer el examen para garan zar que
por lo menos dos de ellos tendrán las mismas respuestas para todas las preguntas?
43. Dados 6 puntos sobre una circunferencia, coloreamos de azul o verde todos los segmentos que
ellos determinan. Demuestre que entre todos los triángulos que quedan formados, hay por lo
menos uno cromá co, es decir, un triángulo con sus tres lados del mismo color.
55
44. Demuestre que si con 2 colores se pintan las diagonales de un pentágono regular, siempre hay un
vér ce del que salen dos diagonales del mismo color.

45. Para una reunión cien fica se han contratado 5 traductores, que deberán cubrir 6 lenguas diferentes.
Si cada una de éstas requiere el empleo de 3 traductores, demostrar que alguno de los intérpretes
deberá traducir por lo menos 4 idiomas.

46. En un cajón hay calce nes negros, rojos, azules y blancos. ¿Cuál es el menor número de calce nes
que hay que sacar para estar seguros de que hay al menos dos del mismo color?

47. En un estadio hay diez mil personas. Demostrar que hay al menos un grupo de 28 personas que
nacieron el mismo día.

48. ¿Cuál es el mayor número de reyes que pueden ser colocados en un tablero de ajedrez de manera
que ninguno dé jaque a ningún otro?

49. A un estadio de fútbol han asis do 37000 espectadores. ¿Cuántos de ellos, como mínimo, cumplen
años el mismo día?

50. Hay 100 personas sentadas en una mesa circular a distancia constante entre sí y al menos 51 de
ellas son mujeres. Verificar que hay al menos 2 mujeres sentados en posiciones diametralmente
opuestas.

51. Considere los primos 2, 3, 5. El conjunto A está formado por los números naturales que se generan
mul plicando dis ntas potencias de estos primos, es decir, los números de la forma

2α 3β 5γ

con α, β, γ enteros no nega vos. Demuestre que de cualquier escogitación de 9 números de A


siempre hay 2 que al mul plicarlos generan un cuadrado perfecto.

52. Suponga que los números del 1 al 10 están ubicados en algún orden sobre una circunferencia.
Prueba que alguna suma de tres números consecu vos suma 17.

53. En un planeta llamado Ω (omega), más de la mitad de la superficie es erra firme. Probar que en
Ω se podría excavar un túnel recto a través del centro del planeta, comenzando y finalizando en
erra firme.

54. ¿Cuántas veces, como mínimo, debe lanzarse un par de dados para asegurarse que el puntaje
obtenido (la suma de los dados) se repita?

55. Sean a, b, c, d enteros, demuestre que (a − b)(b − c)(c − d)(d − a)(a − c)(b − d) es múl plo
de 12.

56. Una prueba de ap tud posee 10 preguntas de selección múl ple, con cinco alterna vas cada una.
¿Cuál debe ser el mínimo número de alumnos que deben dar la prueba (sin dejar respuestas vacías)
para el cual podamos garan zar que por lo menos dos de ellos tendrán exactamente las mismas
respuestas para todas las preguntas?

56
57. En una caja hay 10 libros en francés, 20 en castellano, 8 en alemán, 15 en ruso y 25 en italiano.
¿Cuantos debo sacar para estar seguro de que tengo 12 en un mismo idioma?
58. En un bar hay 95 mesas y un total de 465 sillas. ¿Podemos asegurar que hay una mesa con 6 sillas?
59. Se ene un conjunto de diez números naturales. Demostrar que hay al menos un par cuya diferencia
es múl plo de 9.
60. Demuestre que si del subconjunto de números naturales 1, 2, . . . , 10 extraemos seis números,
con seguridad habrá dos que suman 11.
61. De los números 1, 2, . . . , 100 se toman 51. Demuestre que de estos, hay una pareja que son primos
rela vos, y que hay otra pareja tal que uno divide al otro y el cociente es potencia de 2.
62. Sean a1 , a2 , . . . , a100 y b1 , b2 , . . . , b100 dos permutaciones de 1, 2, 3, . . . , 100. Demuestra que,
entre los productos
a1 b1 , . . . , ai bj , . . . , a100 b100
hay dos con el mismo residuo al dividirse entre 100.
63. Los números 1, 2, . . . , 9 se dividen en tres grupos. Probar que el producto de los números en uno
de dichos grupos, sean cual sean estos, siempre debe ser mayor de 71.
64. En un cubo de lado 10 se colocan 999 puntos. ¿Es posible encontrar siempre un cubo de lado 1
dentro del cubo de lado 10 que no contenga alguno de los puntos?
65. Con los vér ces de una cuadrícula de 6×9 se forman 24 triángulos. Demuestre que hay 2 triángulos
que enen un vér ce en común.
66. Las entradas de una matriz 3 × 3 son los números 0, 1, −1. Probar que entre las ocho sumas que
se ob enen por filas, columnas y diagonales, hay dos iguales.
67. Se enen los números 1, 2, . . . , 2n escritos en una pizarra. Se tachan n1 de ellos. Probar que entre
los números que quedaron sin tachar en la pizarra, hay al menos dos de ellos que son consecu vos.
68. En una pizarra se escriben los números 1, 2, . . . , 2n, probar que si se eligen aleatoriamente n + 1
números de entre ellos, entonces entre los elegidos habrá un par de modo que uno divide al otro.
69. Dados 27 números impares posi vos menores que 100, demostrar que hay al menos dos de ellos
cuya suma es 102.
70. 17 personas se comunican por correo, enviando cada persona una carta a cada una de las demás.
En las cartas sólo son discu das tres temá cas dis ntas. Las cartas enviadas mutuamente entre
dos personas tratan ambas a cerca de una sola de esas temá cas, dado que una persona envía
una carta y la persona a la que va dirigida le responde. Pruebe que hay un grupo de al menos 3
personas tales que en todas las cartas que se enviaron entre sí discu eron a cerca de la misma
temá ca.
71. Algunos de los cuadritos de una cuadricula de 3 × 7 se pintan de negro y los otros se dejan en
blanco. Probar que forzosamente las líneas de la cuadrícula forman un rectángulo en cuyas cuatro
esquinas los cuadraditos enen el mismo color (los cuatro blancos o los cuatro negros).
57
72. Dado un cuadrado de diagonal 3, se marcan al azar 10 puntos. Demostrar que siempre podemos
encontrar al menos dos puntos que están a una distancia no mayor a 1.

73. En un triángulo de área 4 se colocan nueve puntos. Muestre que hay tres de ellos que forman un
triángulo de área menor o igual a 1.

74. Demuestre que un triángulo equilátero de lado 1 no puede ser cubierto totalmente con dos triángulos
equiláteros de lados menores que 1.

75. Un disco cerrado de radio 1cm con ene 7 puntos tales que todas las distancias entre dos de ellos
son mayores o iguales que 1, pruebe que uno de los 7 puntos es el centro del disco.

76. Nos damos un conjunto X de 10 números naturales dis ntos comprendidos entre 10 y 99. Demostrar
que hay dos subconjuntos dis ntos de X tales que la suma de los elementos de los dos subconjuntos
dan el mismo resultado.

77. En un segmento I de longitud 10, inicialmente blanco, se han marcado 10 segmentos disjuntos
con color rojo. Si no hay dos puntos en I a distancia 1 y ambos de color rojo, probar que la suma
de las longitudes de los intervalos es a lo sumo 5.

78. Se dispone de 100 tarjetas, numeradas del 100 al 199. El valor de cada tarjeta es la suma de los
dígitos que aparecen en ella. ¿Cuál es el numero mínimo de tarjetas que hay que extraer para
asegurar que haya tres con el mismo valor?

79. Demostrar que en una fiesta siempre hay dos personas que conocen al mismo número de personas.

80. Comprobar que en una reunión de 6 personas siempre pasa que 3 de ellas se conocen entre sí o
bien 3 de ellas no se conocen entre sí.

81. En una ceremonia de premiación, n personas se saludaron entre sí estrechándose las manos.
Prueba que durante la ceremonia hubo siempre dos personas que estrecharon el mismo número
de manos.

82. En una reunión hay 201 personas de cinco nacionalidades diferentes. Se sabe que en cada grupo de
seis, al menos dos enen la misma edad. Demuestra que hay al menos cinco personas del mismo
país, de la misma edad y del mismo sexo.

83. Demuestre las siguientes iden dades albegraicas:

a)

n
n(n + 1)
i = 1 + 2 + 3 + ··· + n =
i=1
2

b)

n
n(n + 1)(n + 2)
i (i + 1) = 1 · 2 + 2 · 3 + 3 · 4 + · · · + n(n + 1) =
i=1
3

58
c)

n
n(n + 1)(n + 2)(n + 3)
i (i +1)(i +2) = 1·2·3+2·3·4+3·4·5+· · ·+n(n+1)(n+2) =
i=1
4

d) Exprese una iden dad algebraica que generalice las iden dades anteriores y demuéstrela.

84. U lizando las iden dades anteriores, demuestre

a)

n
n(n + 1)(2n + 1)
i 2 = 1 2 + 22 + 32 + · · · + n 2 =
i=1
6

b) ( n )2

n
n2 (n + 1)2 ∑
i 3 = 13 + 2 3 + 3 3 + · · · + n 3 = = i
i=1
4 i=1

c) Encuentre una iden dad algebraica para la suma de las potencias cuartas de 1 a n.

85. Sea S = {1, 2, 3, . . . , n + 1} en donde n ≥ 2, y sea T = {(x, y , z ) ∈ S 3 |x < z, y < z}. Al


contar |T | de dos modos dis ntos, muestre que:


n ( ) ( )
n+1 n+1
k = |T | =
2
+2
k=1
2 3

(3n)!
86. Demuestre que es entero para todo natural n.
2n 3n
87. Sea A = {1, 2, 3, . . . , n}, donde n ∈ N

a) Para k ∈ A, muestre que el número de subconjuntos de A en los que k es el número mayor, es


igual a 2k−1 .
b) Muestre a par r de lo anterior que:


n−1
2i = 2n − 1. (2)
i=0

Lectura Complementaria: Eder Alexander Jacobo. Material de Autoformación Docente para Matemá ca
9° grado. Lección 8. Desarrollo binomial y Mul binomial. MINED

59

You might also like